You are on page 1of 62

Contents of SBI Clerk Prelims 2020 | Capsule | 100% Selection Plan

SBI Clerk Prelims Mini Mocks Schedule............................................................................................................................................ 2

Must Watch Videos Sessions for SBI Clerk Prelims for Quick Revision ................................................................................ 3

SBI Clerk Prelims 2019| Memory Based Paper | For Practice .................................................................................................. 4

SBI Clerk Prelims 2018| Memory Based Paper | For Practice ................................................................................................16

SBI Clerk Prelims 2016| Memory Based Paper | For Practice ................................................................................................27

Previous Year Exam Analysis 2019 and 2018 ..............................................................................................................................37

What should be good Attempts? ........................................................................................................................................................41

Most Important Questions | Quantitative Aptitude | SBI Clerk Prelims 2020 ..................................................................42

Most Important Questions | English Language | SBI Clerk Prelims 2020 ...........................................................................48

Most Important Questions | Reasoning Ability | SBI Clerk Prelims 2020...........................................................................56

1 Adda247 | No. 1 APP for Banking & SSC Preparation


Website: bankersadda.com | sscadda.com | store.adda247.com | Email: contact@bankersadda.com
SBI Clerk Prelims Mini Mocks Schedule

Date Day Quant Schedule English Schedule Reasoning Schedule

Mini Mock 1 | RC + Mini Mock 1 | Linear


Mini Mock 1 | Missing Series +
15th Sentence Puzzle+ Direction+
Saturday Simplification + (Percentage + Ratio
Feb Rearrangement + Syllogism+ Short
& Proportion + Average) | 20Q
Fillers|20 Q Puzzle|20Q

Mini Mock 2| Month


Mini Mock 2 | Wrong Series +
16th Mini Mock 2| Starters based Puzzle + Inequality
Sunday (Mixture & Alligation + Ages +
Feb + Word Usage|15 Q + Alphabet/ Number
Partnership) | 15Q
based Question | 15Q

Mini Mock 3 | Quadratic Inequality


Mini Mock 3| Circular
+ Quantity Based + (Profit & Loss + Mini Mock 3| Cloze
17th Puzzle + Coding-
Monday Simple Interest & Compound Test+ Column Based +
Feb Decoding + Blood
Interest + Speed Time Distance ) | Error Detection|15 Q
Relation | 15Q
20Q

Mini Mock 4 | Approximation + Mini Mock 4|Word Mini Mock 4| Triangular


18th
Tuesday Table DI + (Probability + swap + Spelling + Puzzle+ Alphanumerical
Feb
Permutation & Combination) | 15Q Fillers |15Q Series+ Syllogism| 15Q

Mini Mock 5| Floor


Mini Mock 5 | Caselet + (Time & Mini Mock 5| Error
19th Puzzle+ Direction+
Wednesday Work + Pipe & Cistern + Detection + Para
Feb Inequality+
Mensuration) | 15Q Jumble|15 Q
Miscellaneous| 20Q

Mini Mock 6| Mix Puzzle


20th Mini Mock 6 | DI-Bar + DI-Line + Mini Mock 6 | Fillers + +Blood Relation + Short
Thursday
Feb DI-Pie | 15Q RC |15 Q Puzzle +
Miscellaneous|15Q

Mini Mock 7| Error


Mini Mock 7 | Missing Series + Mini Mock 7| Box Puzzle
21st Detection + Cloze Test
Friday Simplification + Quadratic + Coding-Decoding+
Feb + Fillers + Phrase
Inequality | 15Q Alphanumeric Series|15Q
Rearrangement|20 Q

The Mini Mocks will be live on Bankersadda.com and Adda247 App.


Download the App Now

2 Adda247 | No. 1 APP for Banking & SSC Preparation


Website: bankersadda.com | sscadda.com | store.adda247.com | Email: contact@bankersadda.com
Must Watch Videos Sessions for SBI Clerk Prelims for
Quick Revision
For the revision Purpose, we are sharing the Complete Playlist for SBI Clerk Prelims. This playlist includes
all the Topics for all the Sections i.e Quantitative Aptitude, Reasoning Ability & English Language. These
topics are taken by the top educators from Adda247. So, do subscribe the channel as well so that you can
attend the Live Classes of our team for SBI Clerk Prelims as well.

S.no Name of the Section Link of the Playlist


1. Reasoning Ability Click Here
2. Quantitative Aptitude Click Here
3. English Language Click Here

Follow the Complete Live Sessions Schedule by the Top Faculties, Subscribe
Adda247 You tube Channel Now.

3 Adda247 | No. 1 APP for Banking & SSC Preparation


Website: bankersadda.com | sscadda.com | store.adda247.com | Email: contact@bankersadda.com
SBI Clerk Prelims 2019| Memory Based Paper | For
Practice
REASONING ABILITY

Direction (1-5): Study the following information carefully 6. Who among the following faces Q?
and answer the questions given below: (a) B (b) D (c) A
Eight persons are sitting around a circular table facing to (d) C (e) None of these
the center but not necessarily in the same order. 7. Who among the following sits at the extreme end of the
Two persons sit between Q and P (either from left or right). row?
R sits immediate to the right of Q. One person sits between (a) E (b) T (c) C
R and S, who faces to T. Q and T are not immediate (d) R (e) P
neighbors of each other. W sits 2nd to the left of V. Three 8. Four of the following five are alike in a certain way and
persons sit between U and V. hence they form a group. Which one of the following
1. Four of the following five are alike in a certain way and does not belong to that group?
hence they form a group. Which one of the following (a) Q (b) A (c) T
does not belong to that group? (d) D (e) P
(a) Q-W (b) P-U (c) S-W
9. Who among the following sits 2nd to the right of P?
(d) V-T (e) Q-P
(a) S (b) R (c) Q
2. Who among the following sits immediate right of U? (d) T (e) None of these
(a) W (b) R (c) T
(d) S (e) V 10. What is the position of A with respect to E?
(a) 3rd to the left
3. The number of persons sit between Q and T , when (b) Immediate to the left
counted to right of Q is same as the number of persons (c) 2nd to the left
sit between W and ___, when counted to the left of ___? (d) 3rd to the right
(a) P (b) S (c) T
(e) 2nd to the right
(d) U (e) None of these
11. In the word ‘PRODUCE’, how many pairs of the letters
4. Who among the following faces R?
have the same number of letters between them in the
(a) U (b) V (c) P
(d) W (e) Q given word as they have in the English alphabet series?
(a) Four (b) Two (c) One
5. Who among the following sits 3rd to the right of S? (d) Three (e) More than four
(a) P (b) U (c) Q
(d) T (e) None of these Direction (12-15): In each of the questions below are
given some statements followed by some conclusions. You
Direction (6-10): Study the following information have to take the given statements to be true even if they
carefully and answer the questions given below: seem to be at variance with commonly known facts. Read
all the conclusions and then decide which of the given
Ten persons are sitting in two parallel rows containing five
conclusions logically follows from the given statements
persons in each row such a way that there is an equal
disregarding commonly known facts.
distance between adjacent persons. In the first row, A, B, C,
D and E are seated and all of them are facing north. In the 12. Statements: All Chocolate are Toffee. No Toffee is
second row, P, Q, R, S and T are seated and all of them are Muffin. Only a few Muffin are Dark.
facing south. Therefore, in the given seating arrangement, Conclusions: I. No Chocolate is Muffin.
each member seated in a row faces another member of the II. Some Toffee can never be Dark.
other row. (a) If only conclusion II follows.
E sits 2nd from one of the extreme end of the row. P faces (b) If only conclusion I follows.
the one who sits 2nd to the right of E. D sits 2nd to the left of (c) If neither conclusion I nor II follows.
B, who does not sit at the extreme end. Two persons sit (d) If either conclusion I or II follows.
between S and Q. R sits immediate left of S. C sits next to B. (e) If both conclusions I and II follow.

4 Adda247 | No. 1 APP for Banking & SSC Preparation


Website: bankersadda.com | sscadda.com | store.adda247.com | Email: contact@bankersadda.com
13. Statements: No Party is Gathering. All Summary are 20. As many as persons are buying things before D as
Gathering. Only a few Gathering are Committee. after___?
Conclusions: I. Some Committee can be Summary. (a) E (b) B (c) F
II. Some Party can be Committee. (d) A (e) G
(a) If either conclusion I or II follows.
Direction (21-23): Study the following information
(b) If only conclusion I follows.
carefully and answer the questions given below:
(c) If neither conclusion I nor II follows.
(d) If only conclusion II follows. Point S is 15m west of point R. Point R is 30m south of
(e) If both conclusions I and II follow. point Q. Point P is 20m west of point Q. Point U is 15m
south of point P. Point T is 35 north of point S.
14. Statements: No Market is Home. Only a few Home are
Room. Only a few Room is Vance. 21. If point V is exactly between point Q and R, then how
Conclusions: I. Some Home are Vance. far and in which direction is point U with respect to V?
II. Some Market can never be Room. (a)15m, North-East
(a) If only conclusion I follows. (b) 15m, East
(b) If neither conclusion I nor II follows. (c) 10m, North-west
(c) If either conclusion I or II follows. (d) 20m, West
(d) If only conclusion II follows. (e) 20m, North-East
(e) If both conclusions I and II follow.
22. Four of the following are alike in a certain way, so form
15. Statements: Some Ball are Garden. All Garden are a group. Which of the following does not belong to that
Trade. Only a few Trade are Pump. group?
Conclusions: I. Some Ball are Pump. (a) P, T (b) U, Q (c)R, P
II. Some Garden can be Pump. (d) S, Q (e) U, T
(a) If only conclusion II follows.
(b) If neither conclusion I nor II follows. 23. If point W is in 5m east of point U, then what is the
(c) If either conclusion I or II follows. distance between point W and Point S?
(d) If only conclusion I follows. (a) 5m (b) 15m (c) 25m
(e) If both conclusions I and II follow. (d) 10m (e) 20m

Direction (16-20): Study the following information Direction (24-26): Study the following information
carefully and answer the questions given below: carefully and answer the questions given below:
Seven persons are buying different things. Only one person A certain number of persons are sitting in the row. All of
buy thing between C and A. B is buying thing immediately them are facing towards north. Q sits sixth from the right
before C. Two persons buy things between C and D. C buy of S. T sits forth to left of Q. Only two persons sit between
things after D. E buys thing immediately before D. More Q and P. R sits forth to the left of S. U sits between S and T.
than three persons buy things between E and F. Only three V sits second to the right of U. W is third from any of the
persons buy things between D and G. end. S is eight from the left end of the row. Six persons sit
16. How many persons buy things between A and F? between W and V.
(a) One (b) Two (c) Three 24. How many numbers of persons could sit in the row?
(d) Four (e) More than four (a) 14 (b) 18 (c) 23
17. Who among the following buy thing immediately after (d) 15 (e) 20
F? 25. What is the position of ‘W’ with respect to Q?
(a) D (b) B (c) G (a) Fourth to the right
(d) A (e) No one (b) Fifth to the right
18. Who among the following buy thing immediately (c) Fourth to the left
before G? (d) Eighth to the right
(a) D (b) B (c) C (e) Sixth to the left
(d) A (e) No one 26. If X sits immediate right of V then how many persons
19. How many person buy things before A? sit between X and P?
(a) One (b) Two (c) Three (a) Five (b) Six (c) Four
(d) Four (e)More than four (d) Three (e) None of these

5 Adda247 | No. 1 APP for Banking & SSC Preparation


Website: bankersadda.com | sscadda.com | store.adda247.com | Email: contact@bankersadda.com
27. If it is possible to make only one meaningful word with A@34%ENM$86&LDS#986QYZ17%ROG
the 2nd, 4th, 6th and 7th letters of the word @2IB2U&
‘UNILATERAL’ which would be the second letter of the
word from the right end? If more than one such word 31. Which of the following element is sixth to the left of the
can be formed give ‘Y’ as the answer. If no such word fourteenth from the left end of the given arrangement?
can be formed, give ‘Z’ as your answer. (a) 6 (b) % (c) $
(a) Y (b) N (c) L (d) M (e) None of these
(d) T (e) Z
32. If all the symbols are dropped from the series, which
Directions (28-30): In each of the question, relationships element will be twelfth from the right end?
between some elements are shown in the statements. (a) 9 (b) Q (c) R
These statements are followed by conclusions numbered I
(d) Y (e) None of these
and II. Read the statements and give the answer.
(a) If only conclusion I follows. 33. How many such numbers are there in the given series
(b) If only conclusion II follows. which are immediately preceded by a symbol and
(c) If either conclusion I or II follows. followed by a letter?
(d) If neither conclusion I nor II follows. (a) None (b) One (c) Two
(e) If both conclusions I and II follow. (d) Three (e) Four
28. Statements: A ≥ B ≥ C = D > E ≤ F < G 34. How many such letters are there in the given series
Conclusions: I. E < B II. G > E which are immediately preceded by number and
29. Statements: P ≤ R < T = U; Q ≥ T ≤ S ≥ V immediately followed by a symbol?
Conclusions: I. Q > P II. V < R (a) One (b) Two (c) Three
(d) More than three (e) None of these
30. Statements: L > M = O ≥ P; N ≤ M ≥ S ≥ T
Conclusions: I. T ≤ P II. N < L 35. Find the odd one out?
Directions (31-35): Study the following sequence and (a) N64 (b) D86 (c) Y%8
answer the given questions. (d) R27 (e) 8EL

QUANTITATIVE APTITUDE
Directions (36-40): What will come in the place of 38. 4, 3, 4, 9, 32, ?
question mark (?) in the following number series: (a) 75 (b) 155 (c) 125
(d) 175 (e) 165
36. 11, ?, 16, 21, 29, 41
(a) 12 (b) 14 (c) 15 39. ?, 100, 150, 375, 1312.5
(d) 13 (e) 11 (a) 50 (b) 100 (c) 75
(d) 25 (e) 200
37. 1800, ?, 60, 15, 5, 2.5
40. 0, 6, 24, 60, ?, 210
(a) 300 (b) 600 (c) 120 (a) 130 (b) 170 (c) 90
(d) 240 (e) 360 (d) 120 (e) 150
Directions (41-45): Study the bar chart given below and answer the following questions.
Bar chart shows the number of books read by 4 different persons (A, B, C & D) in 2005 and 2006.

100
90
80
70
60
50 2005
40 2006
30
20
10
0
A B C D

6 Adda247 | No. 1 APP for Banking & SSC Preparation


Website: bankersadda.com | sscadda.com | store.adda247.com | Email: contact@bankersadda.com
41. Find average number of books read by A, C & D in 2005. 54. 35% of 150 × 16 = ? – 22
(a) 64 (b) 70 (c) 75 (a) 865 (b) 932 (c) 864
(d) 60 (e) 56 (d) 862 (e) None of these
55. (3080 + 6160) ÷ ? = 330
42. Find ratio of books read by B & C together in 2005 to
(a) 26 (b) 22 (c) 28
books read by A & D together in 2006.
(d) 29 (e) 18
(a) 15 : 16 (b) 5 : 6 (c) 1 : 5
(d) 4 : 7 (e) 2 : 3 56. Difference of the compound interest received in first
year and second year at 20% per annum at CI is Rs
43. Books read by A & D together in 2005 are what percent
1200 then find the sum?
more than books read by C in 2006?
2 1 2 (a) Rs 25,000 (b) Rs 36,000 (c) Rs 35,000
(a) 46 3 % (b) 54 3 % (c) 25 3 % (d) Rs 24,000 (e) Rs 30,000
1 2
(d) 33 3 % (e) 66 3 % 57. Find the total distance covered by boat in each
44. Books read by A & C together in 2005 are how much upstream and downstream in 7 hours if the speed of
more or less than books read by B & D together in boat in still water and speed of current is 21 km/h and
2006? 3 km/h respectively?
(a) 24 (b) 14 (c) 18 (a) 280 𝑘𝑚 (b) 294 𝑘𝑚 (c) 315 𝑘𝑚
(d) 22 (e) 28 (d) 301 𝑘𝑚 (e) 322 𝑘𝑚

45. Books read by B & C together in 2006 are what percent 58. Ratio of income of A to that of B is 5:9. If expenditure of
3 4
of books read by B in 2005? A is th of his income and expenditure of B is th of his
8 9
(a) 100% (b) 120% (c) 250% income and sum of their saving is Rs 1950 then find the
(d) 200% (e) 160% difference between their income?
Directions (46-55): What will come in place of (?) (a) Rs 900 (b) Rs 1000 (c) Rs 880
question mark in the following questions? (d) Rs 960 (e) Rs 920
17.28÷? 59. A alone can do a work in 12 days while A and B
46. = 200 together can do that work in 7.5 days. Find the time
3.6 × 0.2
(a) 120 (b) 1.20 (c) 12 taken by C alone to do that work if C takes 3 days more
(d) 0.12 (e) None of these than that of B alone to do that work?
47. 486 ÷ ? × 7392 ÷ 66 = 1008 (a) 33 days (b) 30 days (c) 23 days
(a) 54 (b) 55 (c) 52 (d) 27 days (e) 28 days
(d) 53 (e) 51 60. Ratio of base and perpendicular side of a right-angled
2 1 triangle is 3:4 and its base is equal to the side of a
48. 14 % of 4200 ÷ √576 = (? ) 2
square having area 81 cm2. Find the perimeter of the
7
(a) 125 (b) 225 (c) 25 triangle?
(d) 5 (e) 625 (a) 30 cm (b) 36 cm (c) 33 cm
2 5 3 (d) 42 cm (e) 40 cm
49. 7 × 6 × 8 ×? = 90
(a) 1208 (b) 1108 (c) 1008 Directions (61-65): In each of these questions, two
(d) 1128 (e) 1348 equations (I) and (II) are given. You have to solve both the
equations and give answer
50. (0.05 × 6.25) ÷ 2.5 =? (a) if x>y
(a) 12.55 (b) 0.125 (c) 0.115 (b) if x≥y
(d) 1.25 (e) None of these (c) if x<y
51. 1496 ÷ 17 = ?% of 220 (d) if x ≤y
(a) 25 (b) 40 (c) 50 (e) if x = y or no relation can be established between x and
(d) 75 (e) None of these y.

52. (36% of 180) ÷ 0.4 = ? 61. I. x² - 13x + 40 = 0


(a) 160 (b) 164 (c) 166 II. 2y² - y – 15 = 0
(d) 162 (e) 180 62. I. 5x² + 17x + 6 = 0
53. 0.08% of 55500 – 16.4 = ? II. 2y² + 11y + 12 = 0
(a) 26.6 (b) 28 (c) 29.2 63. I. 7x² - 19x + 10 = 0
(d) 30.4 (e) 32 II. 8y² + 2y – 3 = 0

7 Adda247 | No. 1 APP for Banking & SSC Preparation


Website: bankersadda.com | sscadda.com | store.adda247.com | Email: contact@bankersadda.com
64. I. 𝑥 2 − 8𝑥 + 15 = 0 1
68. Retailer sold one article at 33 3 % profit and another at
II. 𝑦 2 − 3𝑦 + 2 = 0 100% profit. Find his overall profit percentage if the
65. I. 3x² –7x + 4 = 0 selling price of both the article is same?
2
II. 2y² – 9y + 10 = 0 (a) 60% (b) 55% (c) 66 %
3
2
66. A person travels half of the distance at the speed of x (d) 75% (e) 56 3 %
km/h and remaining half of the distance at 4x km/h.
69. A mixture has milk and water in the ratio 4: 1. When
Find the value of ‘x’ if the average speed is 36.8 km/h? 50% of the mixture is taken out and replaced by 24
(a) 21 (b) 25 (c) 24 liters of water then the ratio of milk to water in the
(d) 23 (e) 20 mixture becomes 1: 1. Find initial quantity of mixture.
67. A, B and C invested in a ratio of 7: 8: 5 in a business. (a) 80 liters (b) 45 liters (c) 70 liters
They got an annual profit of Rs. 136800. If A and C (d) 60 liters (e) 75 liters
withdrew their amount at the end of 3 months and 7 70. 4 years ago, ratio of Shivam’s age to Deepak’s age was
months respectively. Then find the difference between 2: 3 and ratio of Shivam’s age 4 years ago to Deepak’s
A and C’s share of profit? age 5 years hence is 8: 15. Find present age of Shivam.
(a) Rs. 12,600 (b) Rs. 11,500 (c) Rs. 13,500 (a) 32 years (b) 28 years (c) 40 years
(d) Rs. 10,500 (e) Rs. 13,000 (d) 24 years (e) 36 years

ENGLISH LANGUAGE

Directions (71-78): Read the following passage and they are deployed - along with enhanced security through
answer the following questions based on the given the use of multiple server locations. With all this
passage. Some of the words are highlighted which would considered, it is no surprise that 42% of UK businesses
help you to answer some of the questions given. leverage some kind of cloud service, according to Eurostat.
However, all the perceived benefits of leveraging the cloud
For years, world-wide organisations have become are redundant if organisations come up against barriers to
increasingly excited about the prospect of a cloud-based accessing cloud services. Cloud-based data pipelines still
future. As the dream becomes an ever closer reality for suffer from complexity challenges at the moment, along
many kinds of business and, Forrester predicted that with the lack of visibility into cost and resource usage at
enterprise spending on cloud services is set to surge. IDC the application and user level. The answer to this is
also predicted that global spending on public cloud automation fueled by robust Machine learning training
services and infrastructure would reach $210bn in 2019, models and artificial intelligence. These concepts and the
an increase of 24% from 2018. But one obstacle stands tools that enable them can determine the prerequisites of
create friction and introduce risk: the process of migration. cloud infrastructure, application dependencies,the
As all indications point to a massive shift in data appropriate target cloud instance profiles, and provide
deployments to the cloud, it is more important than ever troubleshooting in real-time.
that the transition from on-premises to Cloud is as risk free
as possible. In today's climate any loss or disruption to data To summarise, the promise of the cloud has created a sense
can have a huge business impact. It’s a complex process, is of excitement amongst enterprises, however, they have
frequently underestimated and many organisations have proceeded to go full steam ahead into adopting a cloud
found there’s lots that can go wrong that can impact the service, without sufficient data to ensure performance
business. service level agreements (SLAs).

Organisations across the globe have found the cloud to be 71. How can we tackle the risks associated with the
an ideal place to run modern data applications due to big process of migration?
data’s elastic resource requirements. Furthermore, with (a) By ensuring that organisations only migrate the
the lack of data talent an ever-looming issue for most apps to the cloud that will thrive in the cloud.
companies today they have been determined to adopt a (b) The surge in the investment in the cloud
cloud-first strategy to ensure business operations are technology.
accessible for a range of employees. (c) The use of predictive power of Artificial
Intelligence.
The cloud offers great promise for developers especially, as (d) The transition from on-premises to Cloud
it can increase the speed at which they develop software (e) None of these
features and increase the resilience of applications once
8 Adda247 | No. 1 APP for Banking & SSC Preparation
Website: bankersadda.com | sscadda.com | store.adda247.com | Email: contact@bankersadda.com
72. Why the promise of the cloud has created a sense of 78. Which of the following words is most OPPOSITE to the
excitement amongst enterprises? word
(a) As it ensures that business operations are “MASSIVE” as given in the passage?
accessible for a range of employees. (a) Prominent
(b) As the enterprise spending on cloud services is (b) Insignificant
(c) Enlarged
surging.
(d) Filthy
(c) Reduction in the cost of operation it may entail.
(e) Vouch
(d) The secure platform it provides to the data it
stores. Directions (79-85): Read each sentence to find out if there
(e) None of these is any error in it. The error, if any will be in one part of the
sentence. The number of that part is the answer if there is
73. How the process of migration stands as an obstacle to no error, the answer is (e).
the cloud technology?
79. It is widely believe (a)/ that the village which is
(a) The expensive cost of providing safety to the data.
situated on the (b)/ eastern coast of India will be wiped
(b) The lack of sound technology in Artificial out (c)/ within a decade. (d)/ No error. (e)
Intelligence.
(c) Complexities of the data migrated to the cloud 80. Yesterday I met (a)/ an old friend (b)/ when I am going
(d) It exposes the companies to the risk of losing (c)/ to the market. (d)/ No error.(e)
important information. 81. The criminal was (a)/ sentenced to the (b)/ death and
(e) None of these was (c) hung for his crime (d)/ No error.(e)

74. Which of the following statement(s) is/are correct in 82. FATF strongly urges (a)/ Pakistan to swiftly complete
context of the passage? its action plan (b)/ by October 2019 to stop (c)/ terror
(1) The cloud technology can help in increasing the funding in the country (d)/ No error.(e)
speed at which the software features are 83. Elon musk is determined (a)/ to be success (b)/ in
developed. whatever field (c)/ he chooses. (d)/ No error.(e)
(2) There is no risk associated with the cloud 84. New purchasing power will increasingly (a)/ come
technology. from Asia and Africa where (b)/ the demographics are
(3) Cloud technology is not a very complex process. still favorable (c)/ for high income growth. (d)/ No
(a) Only (2) error (e).
(b) Both (1) and (2)
85. Not only the (a)/ students but also (b)/ the principal
(c) Only (3) were(c)/ laughing at the joke he cracked.(d)/No error
(d) Only (1) (e).
(d) All are correct
Directions (86-90): In each of the following sentences,
75. What is the meaning of the phrase “full steam ahead” ? there is a blank space. Below in each sentence, there are
(a) Interrupting an activity five words out of which one can be used to fill the blank to
(b) Acting cautiously make the sentence grammatically and coherently correct.
(c) Great increase Find the most appropriate word that fit into the blank
(d) Progressing quickly. contextually.
(e) Prudent progression 86. There is nothing to indicate the building's past, _______
the fireplace.
76. Which of the following words is most similar to the
(a) except (b) included (c) belonged
word “ADOPT” as given in the passage?
(d) barred (e) Foster
(a) mold (b) deduce (c) Embrace
(d) Creative (e) Fluster 87. Two circus elephants that performed together _______
when crossing paths 23 years later
77. Which of the following words is most similar to the (a) happy (b) believe (c) angered
word “ENSURE” as given in the passage? (d) rejoiced (e) greater
(a) Confirm
88. The law _______ farmers not only to save and resow
(b) Paced (multiply) seeds, but also to sell them to other farmers,
(c) Redundant no matter what the original source of the seed is.
(d) Weaken (a) coverts (b) implements (c) emits
(e) Reject (d) permits (e) deserts

9 Adda247 | No. 1 APP for Banking & SSC Preparation


Website: bankersadda.com | sscadda.com | store.adda247.com | Email: contact@bankersadda.com
89. Small farmers must be educated and ______ with proper 93. Which of the following should be the THIRD sentence
incentive structures, to engage with agriculture. after the rearrangement?
(a) fascinated (a) A (b) B (c) C
(b) encouraged (d) D (e) E
(c) discouraged
94. Which of the following should be the FIRST sentence
(d) harvested
after the rearrangement?
(e) invited
(a) A (b) B (c) C
90. Raipur despite being _______ between two rivers, the (d) D (e) E
Krishna and the Tungabhadra, it is a dust bowl in May
95. Which of the following should be the FIFTH sentence
first week.
after the rearrangement?
(a) Surrounded
(a) A (b) B (c) C
(b) Located
(d) D (e) E
(c) Revealed
(d) Settle Directions (96-100): In each the following questions four
(e) None of these. words are highlighted. Choose the option reflecting the
word which is either misspelt or grammatically incorrect.
Directions (91-95): Given sentences are not in their exact
If all the highlighted words are correct, choose option (e)
position. Rearrange them to make a coherent paragraph
i.e. “all are correct” as your answer choice.
and then answer the questions given below.
A. It can even make exercise more fun and productive. 96. A British court has ruled that British arms sales to
Saudi Arabia wear unlawful on Humanitarian
B. Plus, hearing others laugh, even for no apparent
Grounds.
reason, can often trigger genuine laughter.
(a) ruled (b) sales (c) wear
C. To add simulated laughter into your own life, search (d) Grounds (e) All are correct
for laugh yoga or laugh therapy groups. Or you can
97. I'm tyred of hearing politicians making pious
start simply by laughing at other people’s jokes, even if pronouncements about their devotion to the people
you don’t find them funny. (a) tyred (b) hearing (c) pious
D. A Georgia State University study found that (d) devotion (e) All are correct
incorporating bouts of simulated laughter into an 98. It is likely that weaving prospered and partly
exercise program helped improve older adults’ mental relocated towards areas within easy reach of the
health as well as their aerobic endurance. army contractors.
(a) prospered (b) relocated (c) within
E. Believe it or not, it’s possible to laugh without
(d) contractors (e) All are correct
experiencing a funny event—and simulated laughter
can be just as beneficial as the real thing. 99. This cat and mouse tactic was purposely designed to
provoke, inrage and panic the unemployed.
91. Which of the following should be the FOURTH sentence (a) tactic (b) purposely (c) inrage
after the rearrangement? (d) panic (e) All are correct
(a) A (b) B (c) C
(d) D (e) E 100.Indian democracy’s robustness is underscored by
92. Which of the following should be the SECOND sentence high voter turnouts, and large number of candedates
after the rearrangement? in the fray.
(a) A (b) B (c) C (a) robustness (b) turnouts (c) candedates
(d) D (e) E (d) fray (e) No correction required

10 Adda247 | No. 1 APP for Banking & SSC Preparation


Website: bankersadda.com | sscadda.com | store.adda247.com | Email: contact@bankersadda.com
Solutions
REASONING ABILITY
Direction (1-5): 15. (a);

Direction (16-20):

Persons
1. (a); 2. (c); 3. (b); E
4. (c); 5. (b); D
A
Direction (6-10): B
C
G
F
16. (c); 17. (e); 18. (c);
19. (b); 20. (e);

6. (c); 7. (b); 8. (e); Direction (21-23):

9. (a); 10. (d); 21. (d);

11. (c);

Direction (12-15):
12. (b);

22. (c);
13. (e);

14. (b);

11 Adda247 | No. 1 APP for Banking & SSC Preparation


Website: bankersadda.com | sscadda.com | store.adda247.com | Email: contact@bankersadda.com
23. (b); Direction (28-30):

28. (e); I. E < B (True) II. G > E (True)

29. (a); I. Q > P (True) II. V < R (False)

30. (b); I. T ≤ P (False) II. N < L (True)

Direction (31-35):

31. (d); M

Direction (24-26): 32 (d); Y

33. (b); One–@2I

24. (e); 25. (a); 26. (c); 34. (a); One – 2U&

27. (b); One word formed- lent 35. (e); 8EL

QUANTITATIVE APTITUDE

36. (d); 43. (e); Required % =


{(72+48)−72}
× 100
72
200 2
= 3
% = 66 3 %

44. (b); Required difference = (72 + 90) – (78 + 70)


= 162 – 148
= 14
37. (a);
78+72
45. (c); Required % = 60
× 100
150
= 60
× 100 = 250%

46. (d); 17.28 ÷? = 200 × 3.6 × 0.2


17.28
38. (b); ?=
144
= 0.12
486 7392
47. (a); ?
× 66
= 1008
486 1008
=
39. (e); ? 112
486
?= 9
? = 54
1
100 1
40. (d); 48. (e); 700
× 4200 × 24 = (? )2
1
25 = (? )2
? = 625
90×7×8
49. (c); ? = 5
= 1008

72+90+48 0.3125
41. (b); Required average = 50. (b); ? =
3 2.5
= 70 ? = 0.125
60+90
42. (a); Required ratio = 90+70 51. (b);
1496
=
?
× 220
17 100
150
= 160= 15: 16 ? = 40

12 Adda247 | No. 1 APP for Banking & SSC Preparation


Website: bankersadda.com | sscadda.com | store.adda247.com | Email: contact@bankersadda.com
52. (d);
36
× 180 ×
10
=? 62. (e); I. 5x² + 17x + 6 = 0
100 4
? = 162 5x² + 15x + 2x + 6 = 0
5x (x + 3) +2(x + 3) = 0
53. (b); 0.08 × 555 − 16.4 =? x = −3, −
2
? = 44.4 − 16.4 ⇒ ? = 28 5
II. 2y² + 11y + 12 = 0
35
54. (d); × 150 × 16 + 22 =? 2y² + 8y + 3y + 12 = 0
100
? = 840 + 22 = 862 2y (y + 4) + 3 (y + 4) = 0
3
9240 y = −4, − 2
55. (c); = 330
? No relation
? = 28
63. (a); I. 7x² - 19x + 10 = 0
56. (e); Let the sum be Rs 100x 7x² - 14x – 5x + 10 = 0
CI in first year= Rs 20x
7x (x - 2) – 5 (x - 2) = 0
CI in two years= 44% of 100x= Rs 44x 5
CI in 2nd year= 44x-20x= Rs 24x x = 2, 7
ATQ II. 8y² + 2y – 3 = 0
24x-20x=1200 8y² + 6y – 4y – 3 = 0
x=300 2y (4y + 3) – 1 (4y + 3) = 0
Required sum=Rs 30,000 y= ,
−3 1
4 2
57. (b); Speed in upstream=18 km/hr x >y
Speed in downstream= 24 km/hr
Required total distance= (24 + 18) × 7 = 64. (a); I. x 2 − 8x + 15 = 0
294 km ⇒ x 2 − 5x − 3x + 15 = 0
⇒ x(x − 5) − 3(x − 5) = 0
58. (d); Let income of A and B be Rs 5x and Rs 9x ⇒ (x − 3)(x − 5) = 0
respectively ∴ x = 3 or 5
15
Expenditure of A=Rs 8 x II. y 2 − 3y + 2 = 0
Saving of A=Rs x
25 ⇒ y 2 − 2y − y + 2 = 0
8 ⇒ y(y − 2) − 1(y − 2) = 0
Expenditure of B= Rs 4x
⇒ (y − 1)(y − 2) = 0
Saving of B= Rs 5x
∴ y = 1 or 2
ATQ
65 ∴x>y
x = 1950 ⇒ x=240
8
Required difference= Rs 960 65. (c); I. 3x² –7x + 4 = 0
⇒ 3x²– 4x - 3x +4 = 0
59. (c); Let total work be 60 units (LCM of 12 and 7.5) ⇒ (3x – 4) (x -1) = 0
Efficiency of A= 5 units/ day 4
x = or 1
Efficiency of A and B together= 8 units/ day 3
Efficiency of B= 3 units/ day II. 2y² -9y + 10 = 0
Time taken by B alone to do that work=20 days ⇒ 2y² - 4y - 5y + 10 = 0
Time taken by C alone=23 days ⇒ (2y - 5) (y -2) =0
5
⇒ y = 2 or 2
60. (b); Side of the square=9 cm
Perpendicular side of the triangle= 12 cm y>x
Hypotenuse of the triangle=√81 + 144 = √225 = 66. (d); Let the distance be D km
15 cm ATQ
Perimeter of the triangle= 36 cm D
D D = 36.8
( + )
61. (a); I. x² -13x + 40= 0 2x 8x

x² - 5x – 8x + 40 = 0 x=23
x (x -5) – 8 (x - 5) = 0 67. (a); Ratio of their profit sharing
x = 5, 8 A: B: C = 7 × 3 ∶ 8 × 12 ∶ 5 × 7 = 21 ∶ 96 ∶ 35
II. 2y² - y – 15 = 0 Annual profit = 136800
2y² - 6y + 5y – 15 = 0 Difference b/w A and C’s share of profit
2y (y - 3) + 5 (y - 3) = 0 14
y=3, -5/2 = × 136800
152
x>y = Rs 12,600

13 Adda247 | No. 1 APP for Banking & SSC Preparation


Website: bankersadda.com | sscadda.com | store.adda247.com | Email: contact@bankersadda.com
68. (a); Let SP of both article = 8x 15x = 24
ATQ, x = 1.6
So, required quantity = 40x + 10x
= 50x
= 80 liters

Profit % =
16x−10x
× 100 = 60% 70. (b); Let age of Shivam and Deepak 4 years ago be ‘2x
10x years’ and ‘3x years’ respectively.
69. (a); Let initial quantity of milk and water in the ATQ,
2x 8
mixture be ‘40x liters’ and ‘10x liters’ =
3x+4+5 15
respectively. 2x 8
= 15
ATQ, 3x+9
(40x× )
1
1
30x = 24x + 72
2
1
10x× +24
=
1 6x = 72
20x
2
1 x = 12
= So, present age of Shivam = 2x + 4
5x+24 1
20x = 5x + 24 = 28 years

ENGLISH LANGUAGE

71. (c); Refer the last few lines of the 5th paragraph of the 81. (d); Use “hanged” in place of “hung”
passage, “The answer to this is automation fueled
82. (e); There is no error in the sentence.
by robust Machine learning training models and
artificial intelligence……………. provide 83. (b); There is an error in part (b) of the sentence. Use
troubleshooting in real-time.” “to succeed” in place of “to be success” because
72. (a); Refer the 3rd paragraph of the passage, after “determined” we use “to infinitive”.
“Furthermore, with the lack of data talent an 84. (e); There is no error in the sentence.
ensure business operations are accessible for a
range of employees.” 85. (c); Replace 'were' with was'.
When two subjects are joined by 'not only...but
73. (d); Refer to the 2nd line of the 2nd paragraph of the also', the verb must agree with the second subject.
passage, “In today's climate any loss or disruption The correct sentence should be: “Not only the
to data can have a huge business impact.”
students but also the principal was laughing at the
74. (d); Statement (1) is correct. Refer the 4th paragraph joke he cracked.
of the passage, “The cloud offers great promise for The same rule applies when two subjects are
developers the use of multiple server locations” joined by 'or', the verb must agree with the second
75. (c); If something such as a plan or a project goes full subject.
steam ahead, it progresses quickly. For e.g. Krish and his brothers were there.
(Second subject is 'plural') Mohan or Sohan is
76. (c); Embrace means accept (a belief, theory, or responsible for this. (Second subject is 'singular')
change) willingly and enthusiastically hence it is
similar in meaning to adopt. 86. (a); “Except” which means “not including” is the word
which should fit the
77. (a); Ensure means make certain that (something) will
blank.
occur or be the case hence confirm is the word
which is most similar in meaning. 87. (d); “Rejoiced” which means feel or show great joy or
78. (b); Massive means exceptionally large hence delight will fit the blank most appropriately.
insignificant is the word which is most opposite in 88. (d); Here, “permits” which means “officially allow
meaning. (someone) to do something” perfectly fits in the
79. (a); There is an error in part (a) of the sentence in given blank making the sentence grammatically
place of “believe” we will use “believed”. correct and contextually meaningful. Hence,
option (d) is the most suitable answer choice.
80. (c); There is an error in part (c) of the sentence. Coverts: a thicket in which game can hide.
In place of “am” we will use “was” because the Implements: put (a decision, plan, agreement,
events of the sentences are in past.
etc.) into effect.

14 Adda247 | No. 1 APP for Banking & SSC Preparation


Website: bankersadda.com | sscadda.com | store.adda247.com | Email: contact@bankersadda.com
Emits: produce and discharge (something, meaningfully and grammatically correct. Hence,
especially gas or radiation). option (b) is the most suitable answer choice.
Deserts: what a person deserves with regard to
91. (b); The correct sequence for the given sentences is
reward or (more usually) punishment. EADBC.
89. (b); Here, “encouraged” which means “give support, 92. (a); The correct sequence for the given sentences is
confidence, or hope to (someone)” perfectly fits in EADBC.
the given blank making the sentence
93. (d); The correct sequence for the given sentences is
grammatically correct and contextually
EADBC.
meaningful. Hence, option (b) is the most suitable
answer choice. 94. (e); The correct sequence for the given sentences is
Fascinated: strongly attracted and interested. EADBC.
Discouraged: having lost confidence or 95. (c); The correct sequence for the given sentences is
enthusiasm; disheartened. Harvested: gather (a EADBC.
crop) as a harvest.
96. (c); “Were” should be used in place of “wear”
Invited: make a formal or polite request for
(something) from someone 97. (a); Correct Spelling is Tired
90. (b); ‘Surrounded’ should be followed by ‘By’ or ‘with’, 98. (e); All the given words are correct.
hence it is an incorrect option. Also, option (d) is 99. (c); The correct spelling is “enrage”
grammatically incorrect and option (c) doesn’t
100.(c); From the given highlighted words, ‘Candedates’ is
make a meaningful sentence. So, option (b) is the
misspelled, correct spelling is ‘Candidates’ Hence,
only option which makes the sentence both
option (c) is the correct answer.

15 Adda247 | No. 1 APP for Banking & SSC Preparation


Website: bankersadda.com | sscadda.com | store.adda247.com | Email: contact@bankersadda.com
SBI Clerk Prelims 2018| Memory Based Paper | For
Practice
REASONING ABILITY

Direction (1–3): Read the information carefully and law of L. D is the maternal grandfather of P, who is a male.
answer the questions: Q is the only son of W. W is the grandfather of N and C is
the daughter of N.
A company ABC printed different number of books in
different years 1947, 1956, 1987, 1998, 2002 such that 7. How L is related to C?
number of books printed are not same in any year. 66 (a) Mother (b) Son (c) Brother
books were printed in an odd numbered year which is not (d) Father (e) None of these
1947.The number of books printed in 1947 is 10 less than
that printed in 1987. 59 books were printed in an year 8. How is P related to N?
before the year in which 61 books are printed but not (a) Mother (b) Son (c) Brother
immediate before. The number of books printed in 2002 is (d) Father (e) None of these
2 more than that printed in 1998. Direction (9 – 11): Read the information carefully and
1. How many books were printed in 1947? answer the question:
(a) 56 (b) 66 (c) 63 Point U is 10m north of point Q. Point T is 10m east of point
(d) 61 (e) none of these U. Point S is 15m south of point T. Point P is 20m south of
2. What is the difference between the number of books point Q. Point R is 25m east of point P. Point L is 15m east
printed in 1956 and 2002? of point S. Point M is the midpoint of point U and P.
(a) 7 (b) 10 (c) 8 9. What is the distance between point L and R?
(d) 4 (e) none of these (a) 10m (b) 15m (c) 5m
3. In how many years the number of books printed are (d) 20m (e) 25m
more than that printed in 1998?
10. In which direction is point T with respect to P?
(a) two (b) one (c) none
(a) north-west
(d) three (e) four
(b) south-west
4. How many words can be formed from the 1st, 6th, 8th (c) south-east
and 9th letter of a word ‘EMANICIPATE’ by using each (d) north-east
letter once in the word? (e) none of these
(a) two (b) one (c) none
(d) three (e) more than three 11. Which of the following points are inline?
(a) P, R, S (b) Q, M, L (c) U, S, T
5. If all the letters in the word FIGURES are arranged in
(d) M, S, L (e) Q, S, L
alphabetical order from left to right in such a way that
vowels are arranged first followed by consonants, then Directions (12–16): Read the following information
how many letters are there in between U and R after carefully and answer the given questions.
the arrangement? Twelve persons sitting in two rows. D, E, F, K, L and M
(a) two (b) one (c) none sitting in row-1 and facing north. S, T, U, X, Y and Z sitting
(d) three (e) four in row-2 and facing south direction. E sits third from one of
6. If in the number 39682147, 1 is added to each of the the extreme ends. S sits second to the left of the one who
digit which is less than five and 1 is subtracted from faces E. Only three persons sit between S and T. K sits
each of the digit which is greater than five then how somewhere right of M. More than three persons sit
many digits are repeating in the number thus formed? between X and T. F faces one of the immediate neighbours
(a) two (b) one (c) none of T. Z sits second to the right of Y. The one who faces L sits
(d) three (e) four third to the left of U. D faces S.
Direction (7 – 8): Read the information given below and 12. Who among the following faces K?
answer the questions. (a) T (b) S (c) X
All the given members belong to the same the family. J is (d) Y (e) none of these
the brother of L. J is the only son of R. W is the father-in-
16 Adda247 | No. 1 APP for Banking & SSC Preparation
Website: bankersadda.com | sscadda.com | store.adda247.com | Email: contact@bankersadda.com
13. Who among the following faces the immediate I. There are only two persons sit between Sahil and
neighbor of M? Geeta. More than three persons sit to the left of
(a) Z (b) K (c) D Geeta.
(d) L (e) None of these II. Not more than 8 persons can sit in a row. Ravi sits
second to the left of Sahil. Diya sits 6 places away
14. Four of the following five from a group, which among
from Geeta.
the following does not belong to this group?
(a) T, E (b) U, D (c) Y, L 20. What is the code of ‘right’ in a certain code language?
(d) Z, E (e) Z, K I. The code of ‘every right to reject’ is ‘%47 *32 $53
*95’,
15. Who among the following faces the one who sit to the
II. The code of ‘never reject right turn’ is ‘%62 %47
immediate left of Y?
$51 *32’.
(a) U (b) D (c) X
(d) Z (e) none of these 21. Find the number of boys and number of girls in the
row?
16. How many persons sit between M and D?
I. R sits 18th from left end of the row and Y sits 11th
(a) one (b) two (c) three
from the right end of the row. R and Y interchange
(d) five (e) four
their positions, after interchanging the position R’s
Direction (17): Five people A, B, X, Y, and Z live on five position is 20th from left end.
different floors of a building (such as ground floor II. Total 43 students are in the row and all are facing
numbered as 1 and top is numbered as 6). There are three is same direction.
floors between A and B. X lives one of the floors above Y. Directions (22–26): Study the following arrangement
17. Who among the following lives on third floor? carefully and answer the questions given below:
(a) B (b) A (c) X B5R1@EK4F7©DAM2P3%9HIW8*6UJ$V
(d) Z (e) Cannot be determined Q#

18. Which of the following elements should come in a place 22. Which of the following is the fifth to the left of the
‘?’ ? seventeenth from the left end of the above
AB3 CE6 FI10 JN15 ? arrangement?
(a) OT20 (b) TO21 (c) OT21 (a) D (b) W (c) *
(d) TS21 (e) None of these (d) 4 (e) None of these

Directions (19–21): Each of the questions below consists 23. Which of the following is exactly in the middle between
of a question and two statements numbered I and II given D and U in the above arrangement?
below it. You have to decide whether the data provided in (a) % (b) H (c) 9
the statement are sufficient to answer the question. Read (d) 3 (e) None of these
both the statements and 24. Four of the following five are alike in a certain way
Given answer: based on their position in the above arrangement and
(a) If the data in statement I alone are sufficient to answer so form a group. Which is the one that does not belong
the question, while the data in statement II alone are to that group?
not sufficient to answer the question. (a) R1E (b) F7D (c) M23
(b) If the data in statement II alone are sufficient to answer (d) 9HW (e) UJ6
the question, while the data in statement I alone are
not sufficient to answer the question. 25. How many such symbols are there in the above
(c) If the data either in statement I alone or in statement II arrangement each of which is immediately preceded
alone are sufficient to answer the question. by a number but not immediately followed by a
(d) If the data even in both statements I and II together are consonant?
not sufficient to answer the question. (a) None (b) One (c) Two
(e) If the data in both statement I and II together are (d) Three (e) More than three
necessary to answer the question. 26. Which of the following is the tenth to the left end of the
19. Who sits immediate to the left of Ravi, who is sitting in thirteenth from the right end?
row. All the persons who are sitting in a row facing (a) F (b) M (c) @
north direction? (d) % (e) 3

17 Adda247 | No. 1 APP for Banking & SSC Preparation


Website: bankersadda.com | sscadda.com | store.adda247.com | Email: contact@bankersadda.com
Direction (27–28): Read the information carefully and 31. If 2 is subtracted from the second digit of all odd
answer the questions: numbers and 2 is added in the first digit of all even
Eight persons A, B, C, D, E, F, G, H are sitting around a numbers, then which number is lowest number after
circular table facing centre. H faces B. Two persons sit the arrangement?
between F and B. E sits 2nd right to D. F sits 2nd right to C, (a) 218 (b) 732 (c) 491
who is one of the immediate neighbors of G. C is not an (d) 929 (e) None of these
immediate neighbor of B.
27. Who among the following sits 3rd left to F? 32. If third digit of highest number is divided by the first
(a) D (b) C (c) B digit of lowest number, then what will be the resultant?
(d) A (e) none of these (a) 4 (b) 6 (c) 4.5
28. Who among the following faces A? (d) 5 (e) None of these
(a) D (b) C (c) B 33. If all the digits in each number are arranged in
(d) G (e) none of these
increasing order, then which number will be the
Directions (29-30): Study the following information highest number after the rearrangement?
carefully and answer the given questions. (a) 218 (b) 732 (c) 491
In a certain code language,
(d) 563 (e) None of these
‘good key friends’ is coded as ‘xo pe cm’
‘key law found’ is coded as ‘xo og bt’ 34. How many numbers will be there in the given series in
‘data key good’ is coded as ‘tu xo pe’ which addition of first and third digit is greater than
29. Which of the following is the code for ‘good’? second digit?
(a) xo (b) pe (c) tu (a) One (b) Two (c) Three
(d) cm (e) None of these (d) Four (e) None of these
30. Which of the following word is coded as ‘og’?
(a) law (b) good (c) found 35. How many numbers will be there in the given series in
(d) Either (a) or (c) (e) key which difference of first and third digit is greater than
second digit?
Directions (31-35): The following questions are based on
the six three digits numbers given below: (a) One (b) Two (c) Three
563 218 732 491 929 (d) Four (e) None of these

Quantitative Aptitude

36. Value A is three times of value B. If value of C is 62.5% 39. In a mixture the ratio of milk and water is 7 : 5. When
of difference between value of A and B, then value C is 56 litres of water is added to this mixture the ratio of
what percent of B?
milk and water in mixture becomes 7 : 3. Find the
(a) 120% (b) 125% (c) 115%
(d) 110% (e) 105% initial quantity of milk and water?
(a) 144 liters (b) 120 liters (c) 116 liters
37. A is 40% more efficient than B and both can complete
a work in 20 days. A and B start work and do it for eight (d) 140 liters (e) 160 liters
day. If remaining work is complete by C in 24 days,
then find in how many days C will complete work 40. A man invested Rs 8400 at the rate R% on SI for two
alone? years and gets a total interest of Rs. 2016. If he invested
(a) 30 days (b) 24 day (c) 20 days Rs. 600 more amount on half of the previous rate of
(d) 16 days (e) 40 days interest for two years, then find the interest get by
38. The length of rectangle is thrice the side of square. man?
Difference between perimeter of rectangle and square (a) 960 Rs. (b) 840 Rs. (c) 1080 Rs.
is 40 cm. If breadth of rectangle is 8 cm, then find area (d) 1020 Rs. (e) 1040 Rs.
of square?
Directions (41-45): Data given about total number of
(a) 169 cm2 (b) 121 cm2 (c) 196 cm2
students visited three towns, read the data carefully and
(d) 144 cm 2 (e) 256 cm 2
answer the question:

18 Adda247 | No. 1 APP for Banking & SSC Preparation


Website: bankersadda.com | sscadda.com | store.adda247.com | Email: contact@bankersadda.com
There are 400 students visited three towns A,B and C. Out 48. 567.93 + 455.97 - ? = (27.98)2
of total students 20% visited only town A, students visited (a) 120 (b) 180 (c) 160
only town B are 15% less than that of students visited only (d) 220 (e) 240
town A. 18% of total students visited only town C. Student 3
√?
visited town both A & B but not C are 8 less than students 49. 124.99 × 24.99 + 6.02 × 8.98 = (7.99)2
visited only town B, students visited town B & C but not A (a) 6 (b) 8 (c) 4
are half the students visited town A & B but not C. Students (d) 5 (e) 7
visited A & C but not B are half the sum of students visited
4.98
only Town B and only town C together, remaining students 50. √?+ 135.98 = 7.89 of 319.98
visited all the three towns. (a) 3025 (b) 2205 (c) 4096
41. Find the total number of students visited at least two (d) 4098 (e) 4047
town?
51. A boat travels110 km downstream and come back
(a) 180 (b) 106 (c) 160
(d) 140 (e) 120 upstream in total 32 hours, if speed of stream is 62.5%
less than speed of boat in still water then find the speed
42. Total students visited town A is how much more than
of stream?
total students visited town C?
(a) 48 (b) 40 (c) 38 (a) 5 km/hr (b) 3 km/hr (c) 7 km/hr
(d) 36 (e) 30 (d) 8 km/hr (e) 10 km/hr

43. Total students who visited all the three towns is what 52. A and B invested total Rs. 20,000 and start a business.
percent of total students visited town B & C but not A? If ‘B’ gets total profit of Rs. 4500 and ratio between
1
(a) 33 % (b) 14 %
2 2
(c) 16 % profit share of B and total profit is 9 : 16, then find
3 7 3
2 2
amount invested by A if both A and B invested for same
(d) 66 3 % (e) 60 3 %
time period?
44. Total students visited only town C is what percent less (a) 8250 Rs. (b) 8060 Rs. (c) 8450 Rs.
than total students visited only town A? (d) 8560 Rs. (e) 8750 Rs.
(a) 10% (b) 12% (c) 15% 53. A 120 meters long train cross a pole in 4.8 sec, then find
(d) 8% (e) 18% in what time train will cross a platform, whose length
is three times of the length of the train?
45. Find the ratio between total students visited only town
(a) 19.2 sec (b) 18.6 sec (c) 12.8 sec
B & only town C together to students visited only town
(d) 20.4 sec (e) 26.8 sec
A?
(a) 4 : 5 (b) 7 : 4 (c) 7 : 2 54. Four years ago, Tina is 18 years younger than Ruchi. If
(d) 7 : 5 (e) 7 : 3 present total age of Ruchi and Tina is 50 years, then
find the ratio between present age of Tina to present
Direction (46 – 50): What will approximate value come at
age of Ruchi?
the place of question mark:
(a) 9 : 17 (b) 17 : 8 (c) 17 : 9
11.98 ×15.89 + ∛215.98 ?
46. - =0 (d) 6 : 17 (e) 8 : 17
√288.98 51.98 ×2 −52.98
(a) 485 (b) 459 (c) 559 55. A shopkeeper marked up an article 40% above its cost
(d) 594 (e) 694 price and gave two successive discounts of 25% and
1.98 10% respectively. If he made a loss of Rs. 352, then find
47. 12.98 % of 399.98 + √? = 2.93 of 125.98
the cost price of the article?
(a) 1024 (b) 1296 (c) 961
(a) 5600 Rs. (b) 5000 Rs. (c) 6400 Rs.
(d) 900 (e) 848
(d) 6800 Rs. (e) 7200 Rs.

19 Adda247 | No. 1 APP for Banking & SSC Preparation


Website: bankersadda.com | sscadda.com | store.adda247.com | Email: contact@bankersadda.com
Directions (56 – 60): Given below bar graph shows total number of books sold by six book stores P, Q, R, S, T and U on
15 April 2015. Read the data carefully and answer the question:
180
160
140
120
100
80
60
40
20
0
P Q R S T U

56. If ratio between books sold by store R on 15 April, (a) 21 (b) 23 (c) 24
2015 and on 15 May, 2015 is 15 : 21. 75% of total books (d) 19 (e) 27
sold on 15 May, 2015 are comic books, then find total
62. (12)2 + (15)2 + ? % of 200 = (22)2 - 5
books sold on 15 May,2015 which are not comic (a) 55 (b) 45 (c) 35
books? (d) 30 (e) 65
(a) 36 (b) 42 (c) 48
(d) 52 (e) 56 63. 145 % of 180 + ? % of 320 = 741
(a) 180 (b) 130 (c) 120
57. Total books sold by store T on 15 April, 2015 is what (d) 140 (e) 150
percent more than total books sold by store U on 15
April, 2015? 64. 46 ÷ 162 × √16 = ?
(a) 80% (b) 60% (c) 40% (a) 56 (b) 60 (c) 64
(d) 70% (e) 80% (d) 72 (e) 68

58. Find difference between average number of books sold 65. (√529 − √289) × 12 = 136 − ?2
by store R & S on 15 April, 2015 and average number (a) 8 (b) 10 (c) 12
of books sold by store Q & U on 15 April, 2015? (d) 16 (e) 20
(a) 40 (b) 25 (c) 35 66. 45% of 160 + ? % of 180 = 19 % of 900
(d) 56 (e) 60 (a) 50 (b) 55 (c) 65
59. Total books sold by store R on 15 April,2015 is what (d) 45 (e) 35
percent less than total book sold by store T on 15 April, 2 1
67. 13 3 − 7 4 = ? +1 2
1
2015? 11 11 11
1
(a) 22 %
1
(b) 20 %
1
(c) 25 % (a) 5 12 (b) 7 12 (c) 4 12
3 3 3 11 11
1
(d) 28 3 %
1
(e) 33 3 % (d) 9 12 (e) 11 12
18×11+232.5
60. Total books sold by store T on 15 May, 2015 are 20% 𝟔𝟖. =?
17×9−30
more than total books sold by same store on 15 April, (a) 2.4 (b) 4.5 (c) 2.5
2015, then find total number of books sold by T on 15 (d) 3.5 (e) 6.5
May, 2015?
69. 8.4% of 300 + 4. 8% of ? = 102
(a)332 (b) 216 (c) 244
(a) 1200 (b) 800 (c) 600
(d) 264 (e) 316
(d) 1800 (e) 1600
Directions (61-70) : What will come at the place of 70. 734 + ? = 12.8 × 64
question mark: (a) 80.2 (b) 85.2 (c) 84.8
61. 40% of 50% of 60% of 1200 = ? + 53 (d) 89.5 (e) 78.2

20 Adda247 | No. 1 APP for Banking & SSC Preparation


Website: bankersadda.com | sscadda.com | store.adda247.com | Email: contact@bankersadda.com
English Language

Directions (71-75): In the question given below, some with imperialism and colonialism, and more recently with
sentences/phrases are given out of which one maybe or the capitalist world economy. And as the modern
may not be grammatically correct. Find the grammatically educational system cannot be seen as removed or sealed
correct sentence/phrase out of the given options. If all of off from cultural and political-economic processes
the given sentences/phrases are correct, mark option (e) involved in most conceptualizations of globalization, the
i.e. “All are correct” as your answer. impacts of globalization processes upon education are
often considered wide-ranging, though many are also
71. (a) Do I knows you?
controversial.
(b) He gets tough at times.
Major trends: From a functionalist perspective, the
(c) You must has acknowledged.
globalization of educational systems has been influenced
(d) We didn’t knew.
by new demands and desires for educational
(e) All are correct
transferability, of students and educators. In place of
72. (a) Do you know what time it is in Boston? dichotomous systems in terms of academic levels and
(b) Won't somebody please help me? credentialing, curriculum, and assessment, increasing
(c) I quit a long time ago. convergence can be observed today, as it is recognized that
(d) I had to work hard when I was young. standardization makes movement of people in education
(e) All are correct across societies more readily feasible, and that such
movement of people can __________ education in a number
73. (a) She agreed in him that I should go to the meeting. of ways (to achieve diversity, to increase specialization and
(b) To tell the truth, I used to smoking two packs a day. the promotion of dedicated research centers, to enhance
(c) We agrees to leave soon. global employability, and so on). Thus, the mobility and
(d) I hope my last mail didn't sound too harsh. paths of movement of students and academics, for
(e) All are correct education and better life opportunities, have been a rapidly
74. (a) Neither her friends nor Mary were in a talkative expanding area of research.
mood. 76. Which of the following words is similar in meaning to
(b) None of the story he tells is true. associated?
(c) He is a cut above the average college student. (a) chisel (b) innate (c) equivocal
(d) I don't wanted to play this game anymore. (d) eclectic (e) None of these
(e) All are correct
75. (a) Two against one is not a fair fight. 77. Choose the word that appropriately fits in blank given
(b) She accepts criticism from anyone but her parents. in the passage.
(a) distinct (b) channelize (c) create
(c) Lend him as much money as he needs.
(d) enhance (e) influence
(d) Do you think I should go alone?
(e) All are correct 78. Which of the following is the most appropriate title of
the passage?
Directions (76 – 80): Read the following passage carefully
(a) Aspects of contemporary education
and answer the questions given below them. Certain
(b) Political affairs affecting education
words/phrases have been printed in bold to help you
(c) Modern Education
locate them while answering some of the questions. (d) Globalization’s Impact on Education
Global and transnational processes and practices have (e) Achieving diversity among nations
been observed to influence and impact various aspects of
contemporary education within many geographical 79. Which of the following sentences is in accordance with
contexts, and thus the fields of research related to the theme of the passage?
education and globalization are vast: they are not (a) Globalization is understood as a process or
contained simply within one field or subfield, but can be condition of contemporary human life, at the
seen to cross sub-disciplinary borders, in policy studies, broadest level, rather than a single event or
curriculum, pedagogy, higher education studies, activity.
assessment, and more. (b) Globalization in education cannot be merely
As mentioned previously, modern education can itself be described as harmful or beneficial, but depends on
seen as one most basic instance of globalization, connected one’s position, perspective, values, and priorities.
to increased interdependency of communities around the (c) Global citizenship education has been conceived
world in economic and political affairs first associated by political theorists and educational
philosophers.
21 Adda247 | No. 1 APP for Banking & SSC Preparation
Website: bankersadda.com | sscadda.com | store.adda247.com | Email: contact@bankersadda.com
(d) Education held by scholars and political leaders is 86. Despite of the government's protecting, he was the
a key to enhance the modern human condition. victim of an assassination attempt which killed him.
(e) Globalization as a contemporary condition or (a) Despite being the government’s protecting
process clearly shapes education around the globe, (b) Despite being the government’s protection
in terms of policies and values. (c) Despite the government’s protection
(d) Despite of the government’s protection
80. According to the passage, in what way the education
(e) No replacement required
field is vast?
(a) As it is not confined to any geographical region. 87. They feared to spare him lest he would report the
(b) as the impact of globalization on education is wide matter to the king.
ranging. (a) lest he would not report
(c) As it has various aspects that increase the (b) lest he should report
dependency on other communities of the world. (c) lest he will report
(d) both (a) and (b) (d) lest he shall report
(e) All of the above. (e) no replacement required

Directions (81–85): Each of the sentences given below 88. As soon as he finished reading the collect letter, than
contains a blank. Identify the most suitable alternative he flew into a rage and tore it into pieces.
among the five given that fits into the blank to make the (a) than he flown into a rage
sentence logical and meaningful. (b) then he flown into a rage
(c) then he flew into a rage
81. All this at the time seemed merely strange to Pierre: he (d) he flew into a rage
felt he could not _____________its significance. (e) no replacement required
(a) appreciate (b) hold on (c) encrypt
(d) grasp (e) overlook 89. This is perhaps the same car which ran her over
while she was returning from school.
82. History bears several instances of how ambitions of (a) that ran her over while she was
individuals have ruined countries and brought untold (b) which run her over while she was
______________to the people. (c) which run her over when she was
(a) joys (b) injuries (c) miseries (d) that running her over while she was
(d) comforts (e) heartaches (e) no replacement required
83. Labour ______________ is not just good for those on the
90. If you will set your mind to a goal, you will eventually
move but it also has a positive effect on the economy.
achieve it.
(a) mobility (b) constancy (c) durabilty
(a) would set your mind
(d) adaptability (e) resilience
(b) shall set your mind
84. Sometimes it is not only a book but the book read with (c) set your mind
an ________________review of it that gives us an (d) should set your mind
understanding of a reality. (e) no replacement required
(a) alert (b) insightful (c) rational
Directions (91–94): Rearrange the following six
(d) artful (e) obtuse sentences (A), (B), (C), (D), (E) and (F) in the proper
85. An _________________ link exists between democracy and sequence to form a meaningful paragraph then answer the
public freedom. questions given below them.
(a) determinable (b) avoidable (A) The cow made a request to the tiger, “I have got a calf
(c) inextricable (d) involved at home and the calf is very hungry.”
(e) elaborated (B) The tiger was convinced with her plea and allowed her
to return to the village.
Directions (86–90): In the questions given below, there is (C) All the cows became scared and started running
a sentence in which one part is given in bold. The part given towards the village, but one cow was left behind.
in bold may or may not be grammatically correct. Choose (D) The cow said to the tiger, “I will come back after giving
the best alternative among the four given which can my milk to the calf. I promise.”
replace the part in bold to make the sentence (E) Some cows were grazing in a forest. A tiger pounced
grammatically correct. If the part given in bold is already upon them.
correct and does not require any replacement, choose (F) “It is impossible. Because if I leave you now, you will
option (e), i.e. “No replacement required” as your answer. not come back” said the tiger.

22 Adda247 | No. 1 APP for Banking & SSC Preparation


Website: bankersadda.com | sscadda.com | store.adda247.com | Email: contact@bankersadda.com
91. Which of the following will be the SECOND sentence (E) beautiful languages
after rearrangement? (a) EDABC (b) BDACE (c) CABED
(a) A (b) B (c) C (d) CBEDA (e) None of these
(d) D (e) E 97. (A) Go to Boston
92. Which of the following will be the FOURTH sentence (B) Jack could not
after rearrangement? (C) enough money
(a) A (b) D (c) C (D) with us because
(d) F (e) E (E) he didn't have
93. Which of the following will be the THIRD sentence (a) BACED (b) BAEDC (c) BCADE
after rearrangement? (d) BADEC (e) None of these
(a) A (b) B (c) F
(d) D (e) C 98. (A) Sean had to
(B) pumped because
94. Which of the following will be the SIXTH (LAST)
(C) he had accidentally
sentence after rearrangement?
(a) A (b) B (c) D (D) have his stomach
(d) F (e) E (E) eaten some rat poison
(a) AECBD (b) ADBCE (c) ABCDE
Directions (95 – 100): In each question below some
sentences/phrases are given which are named A, B, C, D (d) ADEBC (e) None of these
and E. Rearrange the given five sentences/phrases to make 99. (A) Graduated from our
a coherent sentence/paragraph. The rearranged sequence
(B) English with a native speaker
of the sentences/phrases will be your answer. If after
rearrangement, no option matches, and option (e) i.e. (C) university has studied
“None of these” is given, mark it as answer. (D) for at least two years
(E) every student who has
95. (A) Speaking French
(B) a native speaker (a) DECAB (b) EBCAD (c) EACBD
(C) who hear Tom (d) EADCB (e) None of these
(D) many people 100. (A) About how he
(E) think he is
(a) DCBAE (b) DCEAB (c) DABCE (B) telling Mary the story
(d) DCAEB (e) None of these (C) being eaten by cannibals
(D) Michael remembers
96. (A) That Spanish is
(E) had escaped from
(B) one of the most
(C) some people say (a) AEBDC (b) DBAEC (c) DCAEB
(D) in the world (d) ACEDB (e) None of these

Solutions

REASONING ABILITY
Solutions (1–3) 4. (a); The 1st, 6th , 8th and 9th letter of a word
‘EMANICIPATE’ are E, C, P, A
Years Books
So, the words formed are PACE and CAPE
1947 56
5. (a); The given word is- FIGURES
1956 59 After the rearrangement- EIUFGRS
1987 66 So, the letters between U and R will be two.
1998 61 6. (b);The given number is- 39682147
2002 63 After applied operation- 48573256
So, only digit 5 is repeated in the number thus
1. (a); 2. (d); 3. (a);
formed.
23 Adda247 | No. 1 APP for Banking & SSC Preparation
Website: bankersadda.com | sscadda.com | store.adda247.com | Email: contact@bankersadda.com
Solutions (7–8)

20. (d);right can be coded either %47 or *32.


21. (d);
Solutions (22–26)
22. (a); 5th to left of 17th from left= 17-5= 12th from left = D
23. (c); 9
7. (e);
24. (e); UJ6
8. (c);
25. (d);1@, 3%, 8*
Solutions (9–11):
26. (a);
Solutions (27–28):

9. (b); 10. (d); 11. (d); 27. (e);


Solution (12–16): 28. (d);
Directions (29–30):
Word Code
friends cm
key xo
good pe
12. (d); 13. (a); 14. (d); law/found og/bt
data tu
15. (b); 16. (c);
17. (e); 29. (b);
Floors Persons 30. (d);
5 A/B
Solution (31–35):
4 X/Z
3 X/Z/Y 31. (a); 543 418 932 471 909
2 Z/Y 32. (c); 9/2= 4.5
1 B/A
33. (d);356 128 237 149 299
18. (c); OT21
34. (d);563 218 732 929
19. (e); From both the statements I and II we can find that
35. (b);218 732
Diya sits immediate to the left of Ravi.

24 Adda247 | No. 1 APP for Banking & SSC Preparation


Website: bankersadda.com | sscadda.com | store.adda247.com | Email: contact@bankersadda.com
Quantitative Aptitude
36. (b);Lets A is 3x and B is x = 72 + 30 + 70 + 20 = 192
C = (3x – x)×
62.5 5
= 2x × =1.25x Required difference = 230 – 192 = 38
100 8
1.25x 20 2
Required percent = × 100 = 125% 43. (d);Required percentage = 30 × 100 = 66 3 %
x
80 −72
37. (e); A and B ratio of efficiency = 140x : 100x 44. (a); Required percentage = × 100 = 10%
80
= 7x : 5x 68+72
Total work = 20 (7x + 5x) = 240x units 45. (b);Required ratio = =7:4
80
A and B is eight days = 8 × (7x + 5x) = 96x units 12 ×16 + ∛216 ?
Efficiency of C =
240x −96x 144x
= 24 = 6x units/day 46. (d); - =0
√289 52 ×2 −53
24 192+6 ?
C alone =
240x
= 40 days 17
- 51 = 0
6x 198 ?
− =0
38. (d);Lets length of rectangle is 3x meters and side of 17
594 − ?
51

square x meters = 0 ⇒ ? = 594


51
2(3x + 8) – 4x = 40 2
6x + 16 – 4x = 40 47. (a); 13 × 4 + √? = 3 × 126
2x = 24 ⇒ x = 12 cm 52 + √? = 84
Area of square = a2 √? = 84 − 52
Area of square = 122 = 144 cm2 √? = 32 ⇒ ? = 1024
39. (a); Let milk and water in mixture be 7x and 5x liters 48. (e); 568 + 456 - ? = (28)2
respectively ? = 1024 – 784 ⇒ ? = 240
7x+56 7
= 3
√?
5x 3 49. (b);25 × √125 + 6 × 9 = (8)2
35x – 21x = 168 ⇒ x = 12 liters 3
initial mixture = (7 + 5) × 12 = 144 liters 5 × √? = 64 − 54
3
√? = 2 ⇒ ? = 8
2016 ×100
40. (c); R = 5
8400 ×2 50. (c); √?+ 136 = of 320
R = 12% 8
12 √? = 200 − 136
New rate = = 6%
2
(8400+600) ×6×2
√? = 64 ⇒ ? = 4096
interest = = 1080 Rs.
100 51. (b);Let speed of boat in still water and speed of current
Solutions (41 – 45): be 8x km/hr and
20 3x km/hr respectively
Total students visited only town A = 400 × = 80
100 ATQ –
85
Total students visited only town B = 80 × 100 = 68 110
+
110
= 32
8x+3x 8x−3x
18
Total students visited only town C = 400× = 72 32x = 32 ⇒ x = 1 km/h
100
Total students visited town A & B but not C = 68 -8 Speed of stream = 3 × 1 = 3 km/hr
= 60 52. (e); Let investment of A is Rs. X then investment of B is
Total students visited town B & C but not A Rs. (20,000 – x)
60 16
= 2 = 30 Total profit = 4500 × 9 = 8000 Rs.
Total students visited A & C but not B =
68+72
= 70 ATQ –
2 x 8000−4500
Total students visited all the three towns (20000−x)
= 4500
= 400 – (80 + 68 +72+60+30+70) = 20 16x = 140000 ⇒ x = 8750 Rs.
41. (a); Total number of students visited at least two 53. (a); Let speed of train is S m/s
120
towns S= = 25 m/s
4.8
= 60 + 30 + 70 + 20 = 180 Let train will cross the platform in T sec
120+120×3
42. (c); Total students visited town A 25 =
T
= 80 + 60 + 70 + 20 = 230 25 × T = 480
Total students visited town C T=
480
⇒ T = 19.2 sec
25

25 Adda247 | No. 1 APP for Banking & SSC Preparation


Website: bankersadda.com | sscadda.com | store.adda247.com | Email: contact@bankersadda.com
54. (e); Four years ago – 2 1 3
× 2 × 5 × 1200 = ? +125
5
(T – 4) + 18 = (R −4)
? = 144 – 125 ⇒ ? = 19
R - T = 18 … (i)
Given, R + T = 50 … (ii) ?
62. (a); 144 + 225 + 100 × 200 = 484 - 5
From (i) and (ii)
2R = 68 2 × ? = 479 − 369 ⇒ ? = 55
R = 34 years 145 ?
T = 16 years 63. (e); 100 × 180 + 100
× 320 = 741
16 ?
Required ratio = = 8 : 17 261 + × 320 = 741
34 100
?
55. (c); Let cost price of article = 100x Rs. × 320 = 741 − 261
100
Marked price of article = 140x Rs. 480×100
75 90 ?= ⇒ ? = 150
Selling price of Article = 140x × 100 × 100 320

= 94.5x Rs. 64. (c); (22)6 ÷ (24)2 × 4 = ?


ATQ – 24 × 22 = ? ⇒ ? = 64
100x – 94.5x = 352
65. (a); (23 – 17) × 12 = 136 − ?2
5.5x = 352 ⇒ x = 64 Rs.
Cost price = 64× 100 = 6400 Rs. ?2 = 136 – 72 ⇒ ?2 = 64 ⇒ ? = 8
45 ? 19
56. (b);Total books sold on 15 May,2015 which are not 66. (b); × 160 + × 180 = × 900
100 100 100
comics by store R 72 +
?
× 180 = 171
120 25 1 100
= 15 × 21 × 100 = 168 × 4 = 42 ?
100
× 180 = 171 – 72
180 −100
57. (a); Required percentage = 100
× 100 ?
× 180 = 99
80 100
= × 100 = 80% ?=
99 ×100
⇒ ? = 55
100
180
58. (b);Average number of books sold by store R & S on 15 8−3−6
April, 2015 67. (c); ? = (13 – 7 – 1)+
12
120+90 1 11
= 2 = 105 ? = 5 – 12 ⇒ ? = 4 12
Average number of books sold by store Q & U on 198+232.5
60+100 68. (d); =?
15 April, 2015 = = 80 153−30
430.5
2
?= ⇒ ? = 3.5
Required difference = 105 − 80 = 25 123

180−120 1 8.4 4.8


59. (e); Required percentage = × 100 = 33 3 % 69. (e); 100 × 300 + 100 ×? = 102
180
4.8
60. (b);Total number of books sold by store T on 15 May, ×? = 102 − 25.2
100
120 76.8 ×100
2015 = 180 × 100 = 216 ?= ⇒ ? = 1600
4.8
40 50 60
61. (d);100 × 100 × 100 × 1200 = ? +125 70. (b); ? = 819. 2- 734 ⇒ ? = 85.2

English Language
71. (b);Out of all the given sentences/phrases only the 75. (e); All of the given sentences/phrases are
second sentence is grammatically correct. Hence, grammatically correct. Hence, option (e) is the
option (b) is the correct answer. correct answer.
72. (e); All of the given sentences/phrases are 76. (e); None of the given words is similar in meaning to
grammatically correct. Hence, option (e) is the ‘associated’.
correct answer. 77. (d);The paragraph talks about the impact of
73. (d);Out of all the given sentences/phrases only the globalization on education. Movement of people
fourth sentence is grammatically correct. Hence, across the communities for standardizing
option (d) is the correct answer. education is ultimately improving education.
74. (c); Out of all the given sentences/phrases only the Hence ‘enhance’ is the most appropriate word to
third sentence is grammatically correct. Hence, be filled in the blank making the sentence
option (c) is the correct answer. contextually correct.

26 Adda247 | No. 1 APP for Banking & SSC Preparation


Website: bankersadda.com | sscadda.com | store.adda247.com | Email: contact@bankersadda.com
78. (d);“Globalization’s Impact on Education” is the most 87. (b);The most appropriate phrase to replace the
appropriate title of the passage. incorrect bold phrase in the sentence is “lest he
should report”. It is to be noted that ‘lest’ is
79. (e); Sentence (e) is in accordance with the theme of the followed by ‘should’ and not ‘would’. Therefore,
passage which is summarizing the whole passage. option (b) becomes the most suitable answer
Other sentences are not delivering the exact choice.
explanation. 88. (d);The most suitable phrase that should replace the
phrase given in bold is “he flew into a rage”. It is to
80. (e); All the sentences are correct in context of the be noted that after ‘As soon as, So long as, As long
passage. as’ we do not use ‘than’. Except for option (d) none
81. (d);The most appropriate word that would fill the other options are in absolute grammatical syntax.
blank is ‘grasp’ which means seize and hold firmly. Hence, it becomes the most viable answer choice.
Encrypt means conceal data in (something) by 89. (a); The most appropriate phrase to replace the given
converting it into a code. All the other words do not phrase in bold to make the sentence grammatically
fill the blank appropriately; hence option (d) is the correct is “that ran her over while she was”. It is to
most appropriate choice. be noted that we will use ‘that’ in place of ‘which’
82. (c); The correct option is (c). ’Miseries’ is a plural form after ‘the same’ if the verb is clear. Here the word
‘was’ shows that the verb is clear. Hence,
of noun and it is the only word that fits the blank considering the given rationale option (a) becomes
appropriately. the most suitable answer choice.
Miseries mean feelings of great physical or mental
distress or discomfort. 90. (c); The appropriate phrase to replace the phrase in
bold to make the sentence grammatically correct is
Heartaches are the emotional anguish or grieves, “set your mind”
typically caused by the loss or absence of someone It is to be noted that this sentence is an example of
loved. First Conditional Sentences. In the case of First
83. (a); Option (a) is the correct choice. Conditional Sentences, we use the simple present
Mobility means the ability to move or be moved tense in the if-clause and simple future tense in the
freely and easily. main clause—that is, the clause that expresses the
likely outcome. This is how we indicate that under
Constancy means the quality of being enduring and a certain condition (as expressed in the if-clause),
unchanging. a specific result will likely happen in the future.
Durability means the ability to withstand wear, Hence, considering the given option (c) becomes
pressure, or damage. the most suitable answer choice.
Adaptability means the quality of being able to Solutions (91–94): The proper sequence of sentences to
adjust to new conditions. form a meaningful paragraph will be ECAFDB.
Resilience means the capacity to recover quickly
91. (c); 92. (d); 93. (a); 94. (b);
from difficulties; toughness.
95. (d);The correct sequence of the parts to form a
84. (b);The correct option is option (b).The statement grammatically correct and contextually
talks about an insightful review of a book that gives meaningful sentence is DCAEB. Hence, option (d)
an understanding of a reality. is the most suitable answer choice.
Alert means cautious. 96. (c); The correct sequence of the parts to form a
Insightful means perceptive. grammatically correct and contextually
Rational means based on or in accordance with meaningful sentence is CABED. Hence, option (c) is
reason or logic. the most suitable answer choice.
Artful means clever or skillful, especially in a crafty 97. (d);The correct sequence of the parts to form a
or cunning way. grammatically correct and contextually
Obtuse means annoyingly insensitive or slow to meaningful sentence is BADEC. Hence, option (d)
understand. is the most suitable answer choice.
85. (c); Option (c) is the right choice. 98. (b);The correct sequence of the parts to form a
grammatically correct and contextually
Inextricable means inseparable.
meaningful sentence is ADBCE. Hence, option (b)
Determinable means able to be definitely decided is the most suitable answer choice.
or ascertained.
Elaborated means presented in further detail. 99. (c); The correct sequence of the parts to form a
grammatically correct and contextually
86. (c); The most appropriate phrase to replace the phrase meaningful sentence is EACBD. Hence, option (c) is
given in bold is “Despite the government’s the most suitable answer choice.
protection”. It is to be noted that “despite” does not 100. (b);The correct sequence of the parts to form a
take any preposition with it. It is always followed grammatically correct and contextually
with a noun, pronoun or a gerund. Since option (c) meaningful sentence is DBAEC. Hence, option (b)
is in the precise grammatical syntax, it becomes is the most suitable answer choice.
the most suitable answer choice.
27 Adda247 | No. 1 APP for Banking & SSC Preparation
Website: bankersadda.com | sscadda.com | store.adda247.com | Email: contact@bankersadda.com
SBI Clerk Prelims 2016| Memory Based Paper | For
Practice
REASONING ABILITY

Directions (1-5): To answer these questions study 8. Statements : S > M = Z > T < Q > V
carefully the following arrangement of letters, digits and Conclusions : I. V = S II. Q > M
symbols. 9. Statements : T < U = V  S > P  Q
M 7 Σ 8 L P @ ? 6 N B T Y 3 2 = E $ 4 9 © G H 5. Conclusions : I. S > T II. V  Q
1. How many such letters are there in the arrangement 10. Statements : M  N > R > W, E = J > L  W
each of which is immediately followed by a number? Conclusions : I. E > W II. M > L
(a) Three (b) Four (c) One
Directions (11-15): The following questions are based on
(d) Two (e) None of these the five three – digit numbers given below:
2. How many such symbols are there in the arrangement 684 512 437 385 296
each of which is immediately preceded by a number? 11. If 2 is added to the first digit of each of the numbers
(a) Two (b) Three (c) Four how many numbers thus formed will be divisible by
(d) Nil (e) None of these three?
(a) None (b) One (c) Two
3. If all the symbols are deleted from the arrangement, (d) Three (e) None of these
then which of the following will be fourth to the left of
12. If all the digits in each of the numbers are arranged in
the 17th element from the left end?
descending order within the number, which of the
(a) 9 (b) E (c) 2
following will be the highest number in the new
(d) Y (e) None of these arrangement of numbers?
4. '78' is related to `P ? 6' and ' ?N' is to`T32'in the same (a) 684 (b) 385 (c) 296
way as'2E'is to…….. in the arrangement. (d) 437 (e) None of these
(a)4©H (b)49G (c)4©G 13. What will be the resultant number if the second digit of
(d)9GH (e)None of these the second lowest number is divided by the third digit
of the highest number?
5. If all the numbers are deleted from the arrangement (a) 2 (b) 3 (c) 0
then which of the following will be fifth to the right of (d) 1 (e) 4
the 13th element from the right end?
14. If 1 is added to the first digit and 2 is added to the last
(a) B (b) N (c) Y
digit of each of the numbers then which of the
(d) T (e) None of these following numbers will be the second highest number?
Directions (6-10): In these questions, a relationship (a) 385 (b) 684 (c) 437
between different elements is shown in the statements(s). (d) 296 (e) 512
The statements are followed by two conclusions. Give 15. If in each number the first and the second digits are
answer interchanged then which will be the highest number?
(a) if only conclusion I is true. (a) 296 (b) 512 (c) 437
(b) if only conclusion II is true. (d) 684 (e) 385
(c) if either conclusion I or II is true. Directions (16-17): Study the following information
(d) if neither conclusion I nor II is true. carefully and answer the questions given below:
(d) if both conclusions I and II are true. P is to the north of Q and S is to the east of P, who is to the
south of W. T is to the west of P.
6. Statements : A > B  C < D, C = E > G
Conclusions : I. D > E II. B > E 16. Who among the following is towards south of W and
north of Q?
7. Statements : P  Q > M  N, Q = S (a) P (b) T (c) S
Conclusions : I. S > P II. N < S (d) Q (e) None of these

27 Adda247 | No. 1 APP for Banking & SSC Preparation


Website: bankersadda.com | sscadda.com | store.adda247.com | Email: contact@bankersadda.com
17. W is in which direction with respect to T? 26. Who among the following lives on the topmost floor?
(a) North (b) Northeast (c) Southwest (a) I (b) Q (c) P
(d) West (e) None of these (d) L (e) None of these
Directions (18-22): Study the following information 27. Which of the following combinations is true?
carefully and answer the questions given below: (a) First floor-S (b) Fourth floor-R
Dhondu, Chintu, Titu, Chiku, Sonu, Monu, Bittu and Sonty (c) Third floor-M (d) Sixth floor-I
are sitting around a circular table facing the center. Sonty (e) None of these
is third to the right of Titu and second to the left of Sonu.
Chintu is not an immediate neighbor of Sonty and Titu. 28. How many pairs of letters are there in the word (in
Monu is second to the right of Chiku and is an immediate forward direction) APPLICATION, each of which have
neighbor of Titu. Bittu is not the neighbor of Sonu. as many letters between then in the word as they have
18. Who among the following is second to the right of Titu? between then in the English alphabet?
(a) Sonty (b) Bittu (c) Monu (a) One (b) Two (c) Three
(d) Sonu (e) None of these (d) Four (e) None of these.

19. Who among the following is an immediate neighbor of 29. In a certain coding system, PAPER is written as PERPA
Sonty and Sonu? and SUBJECT is written as JECTSUB, what should be
(a) Dhondu (b) Chintu (c) Titu the code for COUNCIL?
(d) Bittu (e) None of these (a) NCILCOU (b) LICNOUC (c)NCOUCIL
(d) NLICUOC (e) NILCCOU
20. In which of the following pairs the second person is
sitting on the immediate right of the first person? 30. In a certain code language ‘lu ja ka hu’ means ‘will you
(a) Dhondu, Sonty (b) Titu, Chiku meet us’, ‘lu ka hu pa means ‘will you sold us’. Then
(c) Bittu, Sonty (d) Sonu, Sonty What is the code of ‘meet’ in this code language?
(e) Monu, Titu (a) ja (b) lu (c) ka
21. Who among the following is second to the left of (d) hu (e) cannot be determined
Chintu? 31. In a certain code language COMBINE is written as
(a) Titu (b) Sonty (c) Monu XLNYRMV. How will TOWARDS be written in that code
(d) Dhondu (e) None of these language?
22. Who among the following is opposite Chiku? (a) FLDZIWJ (b) GLDZIWH (c) GLEZJWH
(a) Dhondu (b) Bittu (c) Sonty (d) FLEZIWH (e) None of these
(d) Sonu (e) None of these
32. 37 girls are standing in a row facing the school building
Directions (23-27): Study the following information Ayesha is fifteenth from the left end. If she is shifted six
carefully to answer the given questions: places to the right what is her position from the right
Seven neighbours S, P, L, Q, R, M and I live on different end?
floors in the same building having seven floors numbered (a) 16th (b) 21st (c) 20th
one to seven. (The first floor is numbered one, the floor (d) 18 th (e) None of these
above it is numbered two and so on and the topmost floor
is numbered as seven.) 33. X's mother is the mother-in-law of the father of Z. Z is
Three persons live between I and M. M lives on the floor the brother of Y while X is the father of M. How is X
immediately above S, who does not live on an odd- related to Z?
numbered floor. P is neither live on odd number nor (a) Paternal uncle (b) Maternal uncle
topmost floor. I does not live on the first floor. (c) Cousin (d) Grandfather
Two persons live between R and S. Q lives neither on the (e) Brother-in-law
first floor nor on the fourth floor.
34. If A is a brother of B, C is the sister of A, D is the brother
23. Who lives on the floor just above M? of E, E is the daughter of B, F is the father of C. than who
(a) L (b) P (c) Q is the uncle of D?
(d) R (e) None of these (a) A (b) C (c) B
24. How many persons live between L and P? (d) None of these (e) Can’t be determined
(a) None (b) One (c)Two
35. A said to B that B’s mother was the mother-in-law of
(d) Three (e) Can’t be determined
25. Which of the following pairs live on the first floor and A’s mother. How is A’s mother related to B’s mother?
the topmost floor respectively? (a) Daughter-in-law (b) Mother-in-law
(a) L, Q (b) Q, P (c) I, Q (c) Sister (d) Aunt
(d) L, I (e) Can’t be determined (e) Sister-in-law

28 Adda247 | No. 1 APP for Banking & SSC Preparation


Website: bankersadda.com | sscadda.com | store.adda247.com | Email: contact@bankersadda.com
QUANTITATIVE APTITUDE

Directions (36-40): What should come in the place of 44. The total number of Assaults and Murders together in
question mark (?) in the following questions? Bihar is what per cent of the total number of crimes in
7 3 that state?
36. 5 of 58 + 8 of 139.2 =? (a) 29.82% (b) 39.82% (c) 25%
(a) 133.4 (b) 137.2 (c) 127.8 (d) 21.82% (e) 25.5%
(d) 131.6 (e) None of these 45. Find the difference between the number of various
crimes committed in Bihar and that in Rajasthan.
37. 12% of 555 + 15% of 666 =?
(a) 105 (b) 98 (c) 145
(a) 166.5 (b) 167.5 (c) 168.5
(d) 139 (e) 104
(d) 169.5 (e) None of these
Directions (46-50): What should come in the place of
38. 84368 + 65466 − 72009 − 13964 =?
question mark (?) in the following questions?
(a) 61481 (b) 62921 (c) 63861
(d) 64241 (e) None of these 46. 4376 + 3209 – 1784 + 97 = 3125 + ?
(a) 2713 (b) 2743 (c) 2773
39. 337.8 × 331.2 ÷ 335 = 33 × 33? (d) 2793 (e) 2737
(a) 2.8 (b) 3 (c) 3.2
(d) 4 (e) 6 47. √? + 14 = √2601
? 324
(a) 1521 (b) 1369 (c) 1225
40. 529
= ? (d) 961 (e) 1296
(a) 404 (b) 408 (c) 410
48. 85% of 420 + ?% of 1080 = 735
(d) 414 (e) 416
(a) 25 (b) 30 (c) 35
Directions (41-45): Study the following table carefully (d) 40 (e) 45
and answer the given questions: 7 5 1
49. of of of 3024 = ?
The number of various crimes, as supplied by national 3 4 9
crime record, reported in different states in the year (a) 920 (b) 940 (c) 960
2012-13. (d) 980 (e) 840
50. 30% of 1225 – 64% of 555 = ?
(a) 10.7 (b) 12.3 (c) 13.4
(d) 17.5 (e) None of these
51. How many litres of water should be added to a 30 litre
mixture of milk and water containing milk and water
in the ratio of 7 : 3 such that the resultant mixture has
40% water in it?
(a)5 (b)2 (c)3
(d)8 (e) 7
41. The total number of various crimes in HP is
(a) 37803 (b) 38903 (c) 37903 52. The S.I on certain sum of money for 15 months at rate
(d) 36903 (e) 37003 of 7.5% per annum exceed the S.I on same sum at
12.5% per annum for 8 months by Rs 3250 find sum?
42. Find the ratio of Stalking and Assault in UP to Theft and (a)160000 (b)20000 (c)170000
Criminal Trespass in Haryana. (d) 18000 (e) 312000
(a) 28 : 51 (b) 21 : 52 (c) 52 : 21
(d) 14 : 55 (e) 55 : 14 53. 4 men and 3 women finish a job in 6 days, and 5 men
and 7 women can do the same job in 4 days. How long
43. Find the approximate average of Murder and Theft in will 1 man and 1 woman take to do the work?
all the eight states together. 2
(a) 22( ) days
1
(b) 25( ) days
1
(c)5 ( ) days
(a) 1141 (b) 1132 (c) 1311 7
7
2 7

(d) 941 (e) 1021 (d) 12(22) days (e) None of these

29 Adda247 | No. 1 APP for Banking & SSC Preparation


Website: bankersadda.com | sscadda.com | store.adda247.com | Email: contact@bankersadda.com
54. A and B started a business with initial investments in 62. The numerator of a fraction is decreased by 25% and
the ratio 5 : 7. If after one year their profits were in the the denominator is increased by 250%. If the resultant
ratio 1 : 2 and the period for A’s investment was 7 6
fraction is 5, what is the original fraction?
months, B invested the money for
22 24 27
(a) 6 months (b) 2 ½ months (c) 10 months (a) 5
(b) 5
(c) 6
(d) 4 months (e) 7 months (d)
28
(e)
30
5 11
55. An army lost 10% its men in war, 10% of the remaining
due to diseases and 10% of the rest were disabled. 63. What would be the area of a rectangle whose area is
Thus, the strength was reduced to 729000 active men. equal to the area of a circle of radius 7 cm?
Find the original strength. (a) 77 cm2 (b) 154 cm2 (c) 184 cm2
(a) 1000000 (b) 1200000 (c) 1500000 (d) 180 cm 2 (e) 150 cm 2

(d) 1800000 (e) none of these


56. What is the difference between the compound 64. In a village three people contested for the post of
interests on Rs. 5000 for 1 years at 4% per annum village Pradhan. Due to their own interest, all the
compounded yearly and half-yearly? voters voted and no one vote was invalid. The losing
(a)2 (b)3 (c)4 candidate got 30% votes. What could be the minimum
(d)8 (e)none of these absolute margin of votes by which the winning
57. The speeds of John and Max are 30 km/h and 40 km/h. candidate led by the nearest rival, if each candidate got
Initially Max is at a place L and John is at a place M. The an integral per cent of votes?
distance between L and M is 650 km. John started his (a) 4 (b) 2 (c) 1
journey 3 hours earlier than Max to meet each other. If (d) 3 (e) None of these
they meet each other at a place P somewhere between
L and M, then the distance between P and M is : 65. The price of an article is first increased by 20% and
(a) 220 km (b) 250 km (c) 330 km later on the price were decreased by 25% due to
(d) 320 km (e) None of these reduction in sales. Find the net percentage change in
final price of Article.
58. The average weight of boys in a class is 30 kg and the
(a) 20% (b) 18% (c) 38%
average weight of girls in the same class is 20 kg. If the
average weight of the whole class is 23.25 kg, what (d) 10% (e) None of these
could be the possible strength of boys and girls Directions (66–70): What will come in the place of the
respectively in the same class? question mark (?) in the following number series?
(a) 14 and 26 (b) 13 and 27 (c) 17 and 27
(d) 19 and 21 (e) 14 and 27 66. 48, 23, ?, 4.25, 1.125
(a) 10.5 (b) 10 (c) 2.5
59. A profit of 8% is made by selling a shirt after offering a
discount of 12%. If the marked price of the shirt is (d) 11 (e) None of the above
Rs.1080, find its cost price 67. 2, 15, 41, 80, 132, ?
(a) 890 (b) 780 (c) 880 (a) 197 (b) 150 (c) 178
(d) 900 (e) none of these
(d) 180 (e) None of the above
4
60. The difference between of a number and 45% of the
5 68. ?, 15, 75, 525, 4725, 51975
number is 56. What is 65% of the number? (a) 5 (b) 10 (c) 8
(a) 96 (b) 104 (c) 112 (d) 6 (e) None of the above
(d) 120 (e) None of these
69. 4, 19, 49, ?, 229
61. A man can row 24 km upstream and 54 km
downstream in 6 hours. He can also row 36 km (a) 75 (b) 109 (c) 65
upstream and 48 km downstream in 8 hours. What is (d) 169 (e) None of the above
the speed of the man in still water? 70. 840, ?, 420, 140, 35, 7
(a) 18.75 kmph (b) 19.25 kmph (a) 408 (b) 840 (c) 480
(c) 17.65 kmph (d) 15.55 kmph
(d) 804 (e) None of the above
(e)22.75 kmph

30 Adda247 | No. 1 APP for Banking & SSC Preparation


Website: bankersadda.com | sscadda.com | store.adda247.com | Email: contact@bankersadda.com
ENGLISH LANGUAGE

Directions (71-80): Read the passage carefully and (a) Never trust anyone without confirming it yourself.
answer the questions given below it. Certain words/ (b) One who is not hard working is not loved by the
phrases have been given in bold to help you locate them employees.
(c) Good relation with your employees is most
while answering some of the questions.
important in your life.
One day all the employees reached the office and they saw
(d) Your relationship with your friend is the most
a big advice on the door on which it was written: important one that can influence your happiness.
“Yesterday the person who has been hindering your (e) The most important relationship you can have is
growth in this company passed away. We invite you to join the one you have with yourself.
the funeral in the room that has been prepared in the 73. What was inside the coffin?
gym”. In the beginning, they all got sad for the death of one (a) The corpse of the employee who died.
of their colleagues, but after a while they started getting (b) The coffin was empty.
curious to know who was that man who hindered the (c) A note on which there were instructions.
growth of his colleagues and the company itself. (d) There was a mirror inside the coffin.
(e) Photos of every employee.
The excitement in the gym was such that security agents
74. What was the reason of employee’s excitement?
were ordered to control the crowd within the room. The (a) As they were expecting a bonus or promotion.
more people reached the coffin, the more the excitement (b) Because the employee who hindered their growth
heated up. Everyone thought: “Who is this guy who was died.
hindering my progress? Well, at least he died!” One by one (c) As they were happy they will succeed now since no
the thrilled employees got closer to the coffin, and when one will hinder their growth now.
they looked inside it they suddenly became speechless. (d) They were excited because what they found inside
the box was completely opposite of what they
They stood nearby the coffin, shocked and in silence, as if
expected.
someone had touched the deepest part of their soul. There (e) They were curious to know the identity of the man
was a mirror inside the coffin: everyone who looked inside who hindered the growth of his colleagues.
it could see himself.
75. Why everyone was speechless and shocked?
There was also a sign next to the mirror that said: “There is (a) They were shocked as the employee who died was
only one person who is capable to set limits to your the most hardworking one.
growth: it is YOU.” You are the only person who can (b) As they never expected that their growth was
revolutionize your life. You are the only person who can actually hindered by one of their own.
influence your happiness, your realization and your (c) As they found a mirror inside the coffin.
success. You are the only person who can help yourself. (d) As one of the employees died.
(e) They were shocked to find that coffin was empty.
Your life does not change when your boss changes, when
your friends change, when your partner changes, when Directions (76-78): Choose the word/group of words
your company changes. Your life changes when YOU which is MOST SIMILAR in meaning to the word / group of
words printed in bold as used in the passage.
change, when you go beyond your limiting beliefs, when
you realize that you are the only one responsible for your 76. THRILLED
life. “The most important relationship you can have is the (a) Tedious (b) Monotonous (c)Delight
(d) Dull (e) Boring
one you have with yourself.’’
77. INFLUENCE
71. Which of the following is true in context of the passage? (a) Insignificance (b) Domination
(a) The crowd gathered in the office for the funeral. (c) Underwhelming (d) Triviality
(b) The coffin was empty. (e) Weakness
(c) The employee who died was hindering everybody’ 78. REVOLUTIONIZE
growth. (a) Harmony (b) Calm (c) Uprising
(d) Only a person is responsible for his own success. (d) Stagnation (e) Obedience
(e) The crowd loved the employee who died.
72. What can we learn from this passage?
31 Adda247 | No. 1 APP for Banking & SSC Preparation
Website: bankersadda.com | sscadda.com | store.adda247.com | Email: contact@bankersadda.com
Directions (79-80): Choose the word/group of words 86. Suppose, if you (a)/ were left alone to (b)/ live on a
which is MOST OPPOSITE in meaning of the word/ group deserted island(c)/ what would you do? (d)/ No error
of words printed in bold as used in the passage. (e).
79. FUNERAL 87. He wondered that what (a)/would be the next move
(a) Nativity (b) Burial (c) Cremation (b)/of his opponents who had (c)/ vowed to see him
(d) Entombment (e) Inhumation dislodged from power? (d)/No error (e).

80. HINDERED 88. The nation should (a)/ be grateful to (b) the armed
(a) Hamper (b) Inhibit (c) Impede forces for (c)/ protecting it. (d)/No error (e).
(d) Retard (e) Expedite 89. For so many years(a)/it is almost his habit (b)/ to go to
the bed (c)/ at 10 pm daily. (d)/No error (e).
Directions (81-85): Rearrange the following six sentence
(A), (B), (C), (D), (E) and (F) in the proper sequence to form 90. To the men (a)/ who worked so hard (b)/ in the project
a meaningful paragraph and then answer the questions the news was(c)/ profound disappointing. (d)/No
given below. error (e).
A. The man who bought it immediately put it up for Directions (91-95): In each sentence below four words
auction. have been printed in bold which are numbered (a), (b), (c)
B. And he went on to list the many qualities of the animal. and (d) One of these words may be misspelt or
At the end of his sales talk a man said he would give 40 inappropriate in the context of the sentence. Find out the
dinars for it. wrongly spelt or inappropriate word. The number of that
C. "Look at this fine animal!" he shouted to passersby. word is the answer. If all the words are correctly spelt and
"Have you ever seen a better specimen of a donkey? are appropriate the answer is (e) i.e. ‘All correct.'
See how clean and strong it is!"
91. Rising (a)/prices (b)/of food grains will have an
D. Nasruddin Hodja took his donkey to the market place adverse impac (c)/on developing countries. (d)/ All
and sold it for 30 dinars. correct (e)
E. Another man offered 50. A third offered 55.
F. Hodja who was watching was amazed at the interest 92. To deal effectively (a)/with a crisis (b)/quick
everyone was showing in the donkey. decisions (c)/are requited. (d)/All correct (e)

81. Which of the following should be the FIRST sentence 93. The IT Company has succeeded (a)/in achieving high
after rearrangement? growth rate despite (b)/facing (c)/several
(a)F (b)E (c)B (d)/problems. All correct (e)
(d) A (e)D 94. On an average (a)/there are very fern persons
willingly (b)/to take on (c)/such responsibility.
82. Which of the following should be the SECOND sentence
(d)/All correct (e)
after rearrangement?
(a)A (b)B (c)C 95. More than halve (a)/the budget (b)/has been spent
(d)E (e) D (c)/on modernizing (d)/the factory/All correct (e)
83. Which of the following should be the THIRD sentence Directions (96-100): In the following passage, some of the
after rearrangement? words have been left out, each of which is indicated by a
(a)E (b)F (c)D number. Find the suitable word from the options given
(d)C (e)B against each number and fill up the blanks with
appropriate words to make the paragraph meaningfully
84. Which of the following should be the FOURTH complete.
sentence after rearrangement? Jack went to work for a Farmer and (96) a penny. But
(a) B (b) G (c)F while returning home, he dropped it in a brook. At home,
(d) E (e) C his mother (97) him and told him to put his earnings in his
85. Which of the following should be the LAST (SIXTH) pocket the next time.
sentence after rearrangement? The next day, he worked for a cow keeper, who (98) him a
(a)C (b)A (c)B jar of milk. Jack tried to put the jar of milk in his pocket and
(d)D (e)F spilled it everywhere. Once again, his mother rebuked him.
She told him that he
Directions (86-90): Read each sentence to find out should have carried it on his shoulders. The next day, Jack
whether there is any grammatical or idiomatic error in it. was given a donkey. He carried the donkey on
The error, if any, will be in one part of the sentence. The his shoulders. Now, the king had a daughter who never
number of that part is the answer. If there is ‘No error’, the laughed. (99) Jack carrying the donkey on his shoulders,
answer is 5). (Ignore errors of punctuation, if any.) she (100) laughing for the first time.

32 Adda247 | No. 1 APP for Banking & SSC Preparation


Website: bankersadda.com | sscadda.com | store.adda247.com | Email: contact@bankersadda.com
96. (a) Collected (b)Earned (c)Had 99. (a)Detecting (b) Seeing (c)Watching
(d) Accumulated (e)Make
(d)Recognizing (e) Sawing
97. (a) Praised (b)complimented (c) Scolded
(d) lambaste (e)Criticize 100. (a)Start (b)Stopped (c)commence
98. (a) Sold (b)Provide (c) Made (d)Initiated (e) Began
(d) Gave (e)Issued

Solutions

REASONING ABILITY
Direction (1-5): 17. (b);
1. (a): M 7, Y 3, H 5 Direction (18-22)
2. (b):7 Σ , 2 =, 9 ©
3. (b):4th to the Left - 17th from the left=13th from the
left=E
4. (c): 2E….4©G
5. (d):5th to the right – 13th from the right= 8th from the
right=T
Direction (6-10):
6. (a);D > C = E(True) B ≥ C = E(False)
7. (b);S=Q≥P(False) S=Q>M≥N(True)
18. (b); 19. (a); 20. (c);
8. (d);V = S(False) Q > M(False) 21. (d); 22. (c);
9. (a);S ≥ V = U > T(True) V ≥ Q(False) Directions (23-27)
10. (a); E = J > L≥ W(True) M ≥ N > R > W≤ L(False) Floor Person
Direction (11-15): 7 I
6 Q
11. (b);Only 385 will be divisible by 3 when added 2 on
5 R
first digit of each number.
4 P
12. (c); 864 521 743 853 962
3 M
13. (a); 8÷4=2 2 S
14. (e); 786 614 539 487 398 1 L
15. (a); 864 152 347 845 926 23. (b); 24. (c); 25. (d);
Direction (16-17): 26. (a); 27. (c);
28. (a);

29. (a); 30. (a);


16. (a);

33 Adda247 | No. 1 APP for Banking & SSC Preparation


Website: bankersadda.com | sscadda.com | store.adda247.com | Email: contact@bankersadda.com
31. (b);Opposite Letter according to English alphabetical 34. (a);
series.

32. (e; L=15th sifted 6 place to right 15+6=21st R=(37-


21)+1=17
33. (b);
35. (a);

QUANTITATIVE APTITUDE

36. (a); 81.2 + 52.2 = 133.4 42 = 27 + 3𝑥


3𝑥 = 15 ⇒ 𝑥 = 5
37. (a); 66.6 + 99.9 = 166.5
52. (e); Let sum = 𝑥
38. (c); 149834 − 85973 = 63861 15 1 8 1
x× 12 × 7.5 × 100 − 𝑥 × 12.5 × 12 × 100
39. (b); 33?+1 = 337.8+1.2−5
∴?= 9−5−1 = 3 = 3250
3 𝑥
𝑥 − 12 = 3250
40. (d); ?2 = 529 × 324 32
9𝑥−8𝑥
∴? = 23 × 18 = 414 96
= 3250
41. (d); Total No. of crimes in HP = 36903 𝑥 = 96 × 3250
𝑥 = 312000
42. (b); Ratio = 210 : 520
= 21 : 52 53. (a); Let men’s 1 day work = 𝑥
Let women’s 1 day work = 𝑦
2117+14220
43. (e); = 1
4𝑥 + 3𝑦 = 6 ………………(i)
16
16337
= 16 1
5𝑥 + 7𝑦 = 4 ……………..(ii)
≈ 1021 ∴ By solving eqn. (i) and (ii) —
628 1 5
44. (b); Required = × 100 𝑦 = 78 𝑥 = 156
1577
1 1 156 2
= 39.82% ∴Required days = 1 5 = 2+5 = = 22 7
+ 7
78 156 156
45. (c); Required difference = 1577 – 1432 = 145
54. (c); Let B invested money for 𝒙 months.
46. (c); ?= 7682 – 4909 = 2773 ∴ 5×7∶7×𝑥 =1∶2
47. (b); √? = √2601 − 14 = 51 − 14 = 37 ∴ 35 ∶ 7𝑥 = 1 ∶ 2
? = 1369 7𝑥 = 35 × 2
85 𝑥
𝑥 = 10 months
48. (c); 100 × 420 + 100 × 1080 = 735
55. (a); Let initial men = 100
⇒ x = 35 10
Lost in war = 100 × 100 = 10
49. (d); 980 10
Lost in diseases = 100 × 90 = 9
50. (b); ? = 367.5 – 355.2 81
Disables= 100 × 90 = 8.1
= 12.3
∴ Remaining men = 72.9
51. (a); Let Required quantity = 𝑥 When 72.9 remaining total men = 100
21 3
9+𝑥
=2 When 729000 remaining total men = 1000000

34 Adda247 | No. 1 APP for Banking & SSC Preparation


Website: bankersadda.com | sscadda.com | store.adda247.com | Email: contact@bankersadda.com
24 54
56. (a); When compounded yearly, + =6
𝑢 33
Student = 200 𝑢 = 5.5
When compounded half – yearly ∴ Speed of the man in still water
r = 2%, n = 2 33+5.5
= 2 = 2
38.5

 interest = 202 = 19.25 kmph


difference = 202 – 200 = 2 25𝑥
𝑥− 6
57. (d); speed of john = 30 km/hr 62. (d); 100
250𝑦 =5
𝑦+
Speed of max = 40 km/hr 75𝑥
100
6
Let distance b/w p and m = x km =
350𝑦 5
650−𝑥 𝑥
= 40 + 3 75x = 420y
30 𝑥 420
7x = 2240 =
𝑦 75
x = 320 km 𝑥
=
28
𝑦 5
58. (b); Let Boys = 𝑥 22
Girls = 𝑦 63. (b); Required area = ×7×7
7
(30𝑥+20𝑦)
∴ 23.25 = = 154 cm2
𝑥+𝑦
23.25𝑥 + 23.25𝑦 = 30𝑥 + 20𝑦 64. (b); Since winning candidate and his rival got 70% of
6.75𝑥 = 3.25𝑦 total votes.
𝑥 13 ∴ 34 + 36 = 70
=
𝑦 27
Required minimum margin = 36 − 34 = 2
88 100
59. (c); Cost Price = 1080 × × = 880 25×20
100 108 65. (d); Net Change = 20 – 25 – 100
4
60. (b); = 80% =0–5–5
5
= – 10%
(80 – 45) = 35% of the no. = 56
56
65% of the no. = 35 × 65 = 104 66. (a); ÷ 2 – 1 = 23, ÷ 2 – 1 = 10.5, ÷ 2 – 1 = 4.25……
24 54 67. (a); 2 + 13 = 15, 15 + 26 = 41, 41 + 39 = 80, 80 + 52 =
61. (b) 𝑢
+ 𝑣
= 6 … … … . (1) 132
36 48
+ = 8 … … . . (2) ∴ 132 + 65 = 197
𝑢 𝑣
eqn (1) × 3 – eqn (2) × 2 68. (a); 51975 ÷ 11 = 4725, 4725 ÷ 9 = 525,
72
+
162
= 18 525 ÷ 7 = 75, 75 ÷ 5 = 15,
𝑢
72
𝑣
96
15 ÷ 3 = 5
+ = 16
𝑢
66
𝑣 69. (b); 4 + 15 = 19, 19 + 30 = 49, 49 + 60 = 109,
=2 109 + 120 = 229
𝑣
𝑣 = 33
70. (b); 840 ÷ 1 = 840, 840 ÷ 2 = 420, 420 ÷ 3 = 140,
Put in the eqn (1)
140 ÷ 4 = 35, 35 ÷ 5 = 7

ENGLISH LANGUAGE

71. (d);Refer to the last paragraph, ''you are the only 75. (c); Refer to the second paragraph it is explained there
person who can influence your happiness, your that they were shocked to see the mirror.
realization and your success.''
76. (c); Thrill means cause (someone) to have a sudden
72. (e); Refer to the last paragraph, “the most important feeling of excitement and pleasure hence delight is
relationship you can have is the one you have with
most similar in meaning.
yourself.''
73. (d);Refer to second paragraph, ''there was a mirror 77. (b);influence means the capacity to have an effect on
inside the coffin: everyone who looked inside it the character, development, or behaviour of
could see himself''. someone or something, or the effect itself hence
74. (e); they were excited about the identity. domination is the word most similar in meaning.

35 Adda247 | No. 1 APP for Banking & SSC Preparation


Website: bankersadda.com | sscadda.com | store.adda247.com | Email: contact@bankersadda.com
78. (c); Revolutionize means a forcible overthrow of a 86. (a); Remove 'suppose'.
government or social order, in favour of a new 87. (a); Remove 'that'.
system hence uprising is the word most similar in
meaning. 88. (e); No error.

79. (a); Funeral means a ceremony or service held shortly 89. (c); Remove 'the'
after a person's death, usually including the 90. (d);Use ‘profoundly’ in place of ‘profound’.
person's burial or cremation hence nativity is the
91. (c); Change ‘impac’ into ‘impact’
word most opposite in meaning.
80. (e); hindered means make it difficult for (someone) to 92. (d); Change ‘requited’ into ‘required’
do something or for (something) to happen hence 93. (e);
expedite is the word most opposite in meaning.
94. (b); Change ‘willingly’ into ‘willing’
Directions (81-85);The correct sequence is
DACBEF 95. (a); Change ‘halve’ into ‘half’
81. (e); D 96. (b);
82. (a); A 97. (c);
83. (d);C 98. (d);
84. (a); B 99. (b);
85. (e); F 100.(e);

36 Adda247 | No. 1 APP for Banking & SSC Preparation


Website: bankersadda.com | sscadda.com | store.adda247.com | Email: contact@bankersadda.com
Previous Year Exam Analysis 2019 and 2018

Types Of Questions Asked In SBI Clerk Prelims Exams Based on Previous Year Exam
Analysis
SBI Clerk Question Paper: The State Bank Of India has recently released the call letter for the prelims exam
of Junior Associate/ Clerk 2020. Exam is scheduled to take place from 22nd Feb, 29th feb, 1st March and
8th March 2020. Thus, students are only left with limited time to refine their final preparation for the exam.
The best way to do that is to go through the last year question paper in order to get the idea of the difficulty
level of the questions asked and the type of questions asked. Hence, we are providing you with last year
types Of Questions Asked In SBI Clerk Prelims Exams Based on Previous Year Exam Analysis. SBI has
released 8000+ vacancies for the post of Junior Associate (Clerk) in State Bank of India. Although, there is
a huge vacancies but if you are well versed with the scoring areas and the questions pattern, you will
anyhow make through the prelims exam.

SBI Clerk Prelims Exam


SBI Clerk Prelims Exam 2019 was of easy to moderate in level as per the exam analysis of all the shifts
while SBI Clerk Prelims Exam 2018 was moderate in nature. The exam analysis showed a varied trend.

Overall Good Attempts for SBI Clerk Prelims- Previous Years


In 2019, the number of good attempts were around 71-78 out of 100 questions and paper was of easy to
moderate level. This means the cut-off marks went around 60-80 marks depending on the locality.

Sections No. of Questions Good Attempts Level of Difficulty


Numerical Ability 35 22-25 Moderate
Reasoning 35 25-27 Easy to Moderate
English Language 30 24-26 Easy
Total 100 71-78 Easy to Moderate

Overall Good Attempts for SBI Clerk Prelims 2018


In 2019, aspirants were able to make around 63-72 questions out of 100. The level of exam was moderate
and for cut off check the link given below:

The good attempts for 2018 prelims exam is as follows:


Sections No. of Questions Good Attempts Level of Difficulty
Numerical Ability 35 21-24 Moderate
Reasoning 35 22-25 Moderate
English Language 30 20-23 Easy to Moderate
Total 100 63-72 Moderate

37 Adda247 | No. 1 APP for Banking & SSC Preparation


Website: bankersadda.com | sscadda.com | store.adda247.com | Email: contact@bankersadda.com
SBI Clerk Prelims Exam Section-wise Analysis
SBI Clerk Prelims has 3 sections: Numerical ability, Reasoning and English. We have come up with the topic
wise division of each section for your ease. SBI Clerk Prelims 2018 was showing variability. The level of
difficulty seems to be increased here. Candidates should check section wise topics from where the
questions are asked:
Numerical Ability in SBI Clerk Prelims 2019
Generally questions are asked from DI, Simplification & Approximation. So, they are the most important
topics. The question of Numerical Ability are asked from the following topics:
Topics No. of Questions Level
Wrong Number Series 05 Easy-Moderate
Data Interpretation 05 Easy-Moderate
Quadratic Equations 05 Easy
Simplification & Approximation 10 Easy
Arithmetic Word Problem 10 Easy-Moderate
Total 35 Moderate

Numerical Ability in SBI Clerk Prelims 2018


Questions were asked from almost all the topics and the level was moderate. The topic wise questions
asked are given below:
Topics No. of Questions Level
Data Interpretation (Line) 5 Moderate
Simplification 5 Easy
Quadratic Equation 5 Easy
Wrong Number Series 5 Easy-Moderate
Quantity 1, Quantity 2 Questions 5 Moderate
Data Sufficiency 5 Moderate
Arithmetic Word Problems 5 Moderate
Total 35 Moderate

Reasoning in SBI Clerk Prelims 2019


Puzzles and seating arrangement are the areas from where the questions are asked. In SBI Clerk Prelims
2019, there were 4 sets of Puzzles and Seating Arrangement which are given below:–
• Uncertain number of people (Facing North)
• Day Based Puzzle
• Month Based Puzzle (8 people, Arrangement of birthday according to chronological order)
• Parallel Row Linear Seating Arrangement (facing North-South)

38 Adda247 | No. 1 APP for Banking & SSC Preparation


Website: bankersadda.com | sscadda.com | store.adda247.com | Email: contact@bankersadda.com
Topic-wise number of questions with the level is given below:
Topics No. of Questions Level
Puzzles and Seating Arrangement 18 Easy-Moderate
Direction Sense 03 Easy-Moderate
Syllogism 04 Easy
Numeric Series 05 Easy
Coding Decoding 05 Easy
Total 35 Easy-Moderate

Reasoning in SBI Clerk Prelims 2018


Reasoning carries a weighatge of 35 marks in total. Here is the topic-wise breakup:
Topics No. of Questions Level
Data Interpretation (Line) 5 Moderate
Simplification 5 Easy
Quadratic Equation 5 Easy
Wrong Number Series 5 Easy-Moderate
Quantity 1, Quantity 2 Questions 5 Moderate
Data Sufficiency 5 Moderate
Arithmetic Word Problems 5 Moderate
Total 35 Moderate

English Language in SBI Clerk Prelims 2019


Reading comprehension is the area from where you can score well. The questions that are asked are
generally theme, applications and vocabulary related in Reading Comprehension. Other than this, grammar
plays a role in fillers and error detection. Here is the topic-wise question breakup of SBI Clerk Prelims
Exam:
Topic No. of Questions Level
Reading Comprehension 10 Easy-Moderate
Sentence Rearrangement 05 Easy
Spelling Error 06 Easy
Error Detection 04 Easy
Single Fillers (word) 05 Easy-Moderate
Total 30 Easy

39 Adda247 | No. 1 APP for Banking & SSC Preparation


Website: bankersadda.com | sscadda.com | store.adda247.com | Email: contact@bankersadda.com
English Language in SBI Clerk Prelims 2018
English was easier in comparison to other sections. Error detection was the topic from which most of the
questions are asked. Here is the breakup:
Topic No. of Questions Level
Reading Comprehension 6 Moderate
Sentence Rearrangement 4 Moderate
Error detection 10 Easy-Moderate
Phrase Replacement 4 Easy-Moderate
Single Fillers 6 Easy
Total 30 Easy-Moderate

From the exam analysis of SBI Clerk Prelims Exam 2019 and 2018, you must have got an overall idea about
the questions asked in the examination. We have provided you an overall good attempts and topic-wise
break so that you can easily ace through the exam.

40 Adda247 | No. 1 APP for Banking & SSC Preparation


Website: bankersadda.com | sscadda.com | store.adda247.com | Email: contact@bankersadda.com
What should be good Attempts?

What can be the Expected Good Attempts for SBI Clerk Prelims 2020 Exam?

SBI Clerk Prelims Good Attempts 2020: SBI Clerk (Junior Associate) Prelims 2020 will be held on 22 Feb,
29 Feb, 1 March, 8 March, and all the aspirants who are going to appear for the examination must be curious
to know what numbers should they aim for so that they can easily clear the cut-off of preliminary exam and
appear for the next stage i.e. SBI Clerk Mains 2020. The Good Attempts for SBI Clerk Prelims exam will
depend on various factors like:

• The difficulty level of the questions asked in the examination.


• Pattern of the examination (traditional pattern/new pattern).
• Number of vacancies.
• Number of candidates who have appeared in the examination.

Considering the vacancies for this recruitment process and average attempts done by candidates appearing
throughout the nation. But considering the recent examinations of clerical or assistant level, we can make
a certain expectation of what the good attempts for SBI Clerk Prelims exam should be? This will help you
to aim to score accordingly in each section and overall in the exam.

SBI Clerk Prelims 2020 Expected Good Attempts


According to the analysis & current trends of recently held prominent banking exams, we are expecting the
SBI Clerk Prelims exam to be of a Easy-Moderate Level. This means that if you are preparing to crack SBI
Clerk Prelims then you should aim to attempt 71-82 questions with good accuracy.

41 Adda247 | No. 1 APP for Banking & SSC Preparation


Website: bankersadda.com | sscadda.com | store.adda247.com | Email: contact@bankersadda.com
Most Important Questions | Quantitative Aptitude |
SBI Clerk Prelims 2020
Directions (1-6): What should come in the place of the 12. On selling an article on Rs. 1728 the shopkeeper faces
question mark (?) in following number series problems? 28% loss on the cost price. Find out selling price of the
article if its sell on 15% profit?
1. 2, 4, 12, 24, 72, 144, ?
(a) Rs. 2670 (b) Rs. 2340 (c) Rs. 2760
(a) 410 (b) 423 (c) 432
(d) Rs. 2820 (e) Rs. 2450
(d) 256 (e) 620
13. Veer and Anurag sell an article at 20% profit but Veer
2. 4, 9, 25, 49, 121, 169, ? calculates his profit on selling price whereas Anurag
(a) 310 (b) 289 (c) 232 calculates on cost price if cost price of both articles is
(d) 256 (e) 320 same then find out the ratio of selling price of Anurag
3. 2, 3, 5, 9, 17, 33, 65, 129, ? and Veer?
(a) 210 (b) 257 (c) 232 (a) 24:25 (b) 23:24 (c) 1:1
(d) 256 (e) 280 (d) 6:5 (e) 5:6

4. 2, 5, 10, 17, 26, 37, ? 14. Two men undertake a job for Rs. 960, they can
(a) 60 (b) 57 (c) 42 complete it in 16 days and 24 days respectively. They
(d) 50 (e) 80 work along with a third man and take 8 days to
complete it. Then the share of the third man should be
5. 648, 216, 108, 36, 18, 6, ? (a) Rs. 155 (b) Rs. 165 (c) Rs. 160
(a) 6 (b) 5 (c) 3 (d) Rs. 150 (e) Rs. 140
(d) 2 (e) 4
15. A and B enter into a partnership with their capitals in
6. 4, 5, 12, 39, 160, ? the ratio 3 : 5. At the end of 4 months, A withdraws his
(a) 840 (b) 845 (c) 694 capital. If they receive the profits in the ratio 4 : 5, find
(d) 796 (e) 805 how long B’s capital was used?
(a) 2 months (b) 5 months (c) 4 months
Directions (7-11): What should come in the place of the
(d) 3 months (e) 6 months
question mark(?).
1
16. Veer invested rd of his total investment at 5% and
7. 30% of 80 + 45% of 120-50% of 60 = ? 3
1
(a) 88 (b) 48 (c) 78 th at 4% and rest of money at 3% per annum simple
5
(d) 108 (e) 68 interest for the one year and received total interest of
3 1 Rs 290. find the total sum invested by Veer?
8. (−256) × √2744 × (− ) = 2?
224 (a) Rs.15000 (b) Rs.7500 (c) Rs.10000
(a) 1 (b) 3 (c) 5 (d) Rs.12000 (e) Rs.18000
(d) 4 (e) 2
17. Rs 16000 become Rs 24334 in 3 years at the rate of
3
9. of 550 + √?3 = 338 R% per annum of compound interest. Find the value
5
(a) 4 (b) 8 (c) 2 of R?
(a) 10 (b) 15 (c) 20
(d) 6 (e) 16
(d) 12 (e) 18
6 1
10. 42 7 % of 280 ÷ 8 ÷ 4 =? 18. In a party Sanjay mixed two type of liquor in a glass,
(a) 30 (b) 40 (c) 80 type A liquor contains 40% of alcohol and type B
(d) 90 (e) 60 liquor contains 37% of alcohol. Sanjay takes 15 ml
2 3 1 1 from type A liquor and 20 ml from type B liquor, then
11. 5 3 + 12 5 − 7 4 = 10 5 +? find percentage of alcohol in the glass?
60 4 49 4 6 4
(a) (b) 1 (c) (a) 36 7 % (b) 37 7 % (c) 38 7 %
49 5 60
4 43 1 2
(d) − 5 (e) 60 (d) 37 7 % (e) 38 7 %

42 Adda247 | No. 1 APP for Banking & SSC Preparation


Website: bankersadda.com | sscadda.com | store.adda247.com | Email: contact@bankersadda.com
19. A milkman have mixture of 180 ℓ, which contains milk 29. A train goes from Delhi to Mumbai at the speed of 40
and water in the ratio of 5 : 4. Milkman sold 36ℓ of kmph and comes back at the speed of 50 kmph. Find
mixture and added 6ℓ water in remaining mixture out the average speed of train?
250 350 200
and again sold 60ℓ of mixture. Find milk percentage (a) 7 kmph (b) 7 kmph (c) 9 kmph
320 400
in final mixture? (d) kmph (e) kmph
1 7 4 7 9
(a)52 9 % (b)52 9 % (c)53 9 %
30. Two dice throw together. What will be the probability
1 2
(d) 53 3 % (e) 54 3 % of getting a sum less than 12?
35 31 31
(a) (b) (c)
Directions (20-24): In these questions, two equations 36
11
35
7
36
(d) 18 (e) 9
numbered I and II are given. You have to solve both the
equations and give answer: 31. Two dice throw at random, find out the probability of
(a) if x < y getting same digit in both dice?
5 2 1
(b) if x > y (a) 6 (b) 3 (c) 3
(c) if x ≤ y (d)
1
(e)
1
(d) if x ≥ y 2 6

(e) if relationship between x and y cannot be determined 32. In how many ways can letters of word REQUEST be
arranged so that vowels always put together?
20. I. x² – 9x + 18 = 0 (a) 360 (b) 420 (c) 560
II. 5y² – 22y + 24 = 0 (d) 180 (e) 240
21. I. 6x² + 11x + 5 = 0 33. There are 18 players consist of 10 batsmen and 8
II. 2y² + 5y + 3 = 0 bowlers. In how many ways can a team of 11 players
of 6 batsmen and 5 bowlers is to select for a match?
22. I. x² + 10x + 24 = 0 (a) 5880 (b) 11760 (c) 7056
II. y² – √625 = 0 (d) 14112 (e) 8460

23. I. 10x² + 11x + 1 = 0 Directions (34-39): Read the passage given below and
answer the following questions.
II. 15y² + 8y + 1 = 0
There are some data given by a college campus of total 150
24. I.15x² – 11x + 2 = 0 students. 30 students like English, 50 students like Math
II. 10y² – 9y + 2 = 0 and 40 students like Science. 5 students like both English
and Math but not Science, 20 students like both Math and
Direction (25-27): What approximate value should come
Science but not English and 10 students like both Science
in the place of question (?) marks in the given question: and English but not Math. 5 students like all three subjects.
25. 270.05% of 19.99 + 29.95 × 2.01 = ? × 2 Rest of the students like none of three subjects.
(a) 51 (b) 53 (c) 57 34. Number of students who like only English is what
(d) 60 (e) 62 percentage more than that of students who likes only
3
Science?
26. √27.03 × 64.07 + √63.93 × 6.26 = √?× 4 (a) 125% (b) 100% (c) 50%
(a) 16 (b) 256 (c) 4 (d) 200% (e) 150%
(d) 512 (e) 216 35. Find out the ratio of number of students who like all
3 23.95×7.06 the subjects to who don’t like any subjects?
27. 16.09 × √215.99– = √?+ √676.87
2.93×2.01 (a) 1:6 (b) 6:1 (c) 1:15
(a) 1676 (b) 1324 (c) 1764 (d) 2:5 (e) 1:3
(d) 1729 (e) 1024
36. Find out the total number of students who likes only
28. A train covers a certain distance in certain time, if its two subjects?
1 7 (a) 28 (b) 15 (c) 25
speed is increased by 12 2 % it takes 1 18 hours less
(d) 45 (e) 35
time. Find out the distance if its later speed is 72
37. What is the ratio of number of students who don’t like
kmph?
math to who like math as a subject?
(a) 740 km (b) 820 km (c) 640 km (a) 5:1 (b) 2:1 (c) 3:1
(d) 720 km (e) 800 km (d) 1:2 (e) 1:5

43 Adda247 | No. 1 APP for Banking & SSC Preparation


Website: bankersadda.com | sscadda.com | store.adda247.com | Email: contact@bankersadda.com
38. If students who like math increased by 20% when (a) 36% (b) 40% (c) 56%
students who don’t like any of three subjects starts (d) 28% (e) 60%
liking math, then find out the total number of students 39. Find out the average number of students who like
who like math is what percentage of total students in only math and who like Science?
(a) 60 (b) 50 (c) 40
the college?
(d) 30 (e) 55
Directions (40-44): The following Bar graph shows the number of different types of items sold by a shopkeeper in three
months. Read the given graph carefully and answer the following questions.

1000
900
800
700
600
500
400
300
200
January February March

Mouse Keyboard Monitor Printer

40. What is the ratio between sum of total Mouse and 45. Two years ago, A was four times as old as B. 8 years
printer together sold in January to sum of total hence, A’s age will exceed B’s age by 12 years. The
Keyboard and Printer together sold in March? ratio of the present ages of A to B is
(a) 21:23 (b) 23:24 (c) 25:24 (a) 5:3 (b) 3:2 (c) 2:1
(d) 3:4 (e) 2:3 (d) 3:1 (e) 1:3
41. Due to faulty parts, 20% Mouse, 15% Monitors, 10% 46. If the price of a commodity is increased by 18% then
Keyboards and 12% Printers (of total sold in January) by how much percent one should decrease his
were returned by the customers. find no. of total consumption in order to keep expenditure same as
returned items. previous?
(a) 264 (b) 227 (c) 317 900
(a) 59 %
800
(b) 59 %
700
(c) 59 %
(d) 327 (e) 307 600 100
(d) % (e) %
42. What is the difference between total sold items in 49 9

February and in March? 47. In an examination a student who gets 20% of the
(a) 450 (b) 550 (c) 650 maximum marks fails by 5 marks. Another student
(d) 100 (e) 350 who scores 30% of the maximum marks gets 20
43. Total Printer sold in all three months together is what marks more than the pass marks. The necessary
percentage more or less than total Monitor sold in all percentage required for passing is
three months by shopkeeper? (a) 32% (b) 23% (c) 22%
(a) 16% (b) 12% (c) 20% (d) 20% (e) 24%
(d) 17% (e) 14% 48. One year ago, the ratio between Laxman’s and Gopal’s
44. If selling price of Mouse, Keyboard, Monitor and salary was 3 : 4. The ratios of their individual salaries
Printer is in the ratio of 1:2:4:3 in February, then find between last year’s and this year’s salaries are 4 : 5
the revenue of Keyboard is what percentage of that of and 2 : 3 respectively. At present the total of their
Monitor in February? salary is Rs. 4160. The salary of Laxman, now is:
(a) 75% (b) 110% (c) 50% (a) Rs. 1040 (b) Rs. 1600 (c) Rs. 2560
(d) 90% (e) 85% (d) Rs. 3120 (e) Rs. 4210

44 Adda247 | No. 1 APP for Banking & SSC Preparation


Website: bankersadda.com | sscadda.com | store.adda247.com | Email: contact@bankersadda.com
49. Average of marks obtained by Ram in 4 different 50. If the perimeter of a square is equal to the radius of a
subjects was 75. But later it was found that marks of
circle whose area is 2464 sq cm. what is the area of
two subjects were written incorrectly as 69 and 86 in
place of 96 and 68 respectively. Find the correct the square?
average of marks? (a) 49 sq cm (b) 64 sq cm (c) 25 sq cm
(a) 85.50 (b) 82 (c) 77.25
(d) 72.25 (e) 73.75 (d) 196 sq cm (e) 81 sq cm

Solutions
1. (c); The pattern of the series is ×2, ×3 in alternative 13. (a); Let cost price of the article is Rs. X.
terms 5
Veer’s selling price = 𝑅𝑠. 4 𝑥
so, result will be 144×3=432 6
Anurag’s selling price = 𝑅𝑠. 5 𝑥
2. (b); The pattern of the Series be the Square of prime
Required ratio = 24: 25
numbers
(2, 3, 5, 7, 11, 13, 17) 14. (c);
4, 9, 25, 49, 121, 169, 289
Required results= 289
3. (b); The pattern of the series = +1, +2, +4, +8, +16,
+32, +64, +128 1
So, 129+128=257 Share of Third man = × 960 = Rs. 160
6

4. (d); The pattern of the series is +3, +5, +7, +9, +11, 15. (d); Let A’s capital = 3𝑥
+13….. B’s capital = 5𝑥
Let required time be T.
5. (c); The pattern of the series is ÷3, ÷2 alternately.
Ratio of their profit = (4 × 3𝑥) ∶ (𝑇 × 5𝑥)
12𝑥 4
6. (e); The pattern of this series ∴ 5𝑇𝑥 = 5
× 1 + 1,× 2 + 2,× 3 + 3 ,× 4 + 4,× 5 + 5 … .. 3=𝑇
So, ?= 805
∴ Required time = 3 months
30 45 50
7. (b); × 80 + × 120 − × 60 =? 16. (b); Let total investment be Rs.15x.
100 100 100
24 + 54 − 30 =? ATQ,
? = 48 5𝑥×5×1 3𝑥×4×1 7𝑥×3×1
+ + = 290
100 100 100
8. (d);
−256×14
= 2? x= 500
−224
Total Investment by Veer = Rs. 7500
2? = 16
?= 4 17. (b); ATQ,
𝑅
9. (a); 330 + √?3 = 338 24334 = 16000(1 + )3
100
𝑅 24334
√?3 = 8 (1 + )3 =
100 16000
?3 = 64 100+𝑅 3 12167
?= 4 100
= √ 8000
300 1 R=15%
10. (e); 700
× 280 × 8 × 4 =?
? = 60 18. (e); Total quantity of glass = 35 ml
40
2 3 1 1 So, alcohol from type A liquor = 15 × 100
11. (c); 5 + 12 − 7 − 10 + [3 + 5 − 4 − 5] =?
= 6 ml
40+36−15−12 37
60
=? Alcohol from type B liquor = 20 × 100
49
? = 60 = 7.4 ml
100 115
Total alcohol = (6 + 7.4) ml = 13.4 ml
12. (c); Required results = 1728 × 72
× 100 = Rs. 2760 Required % =
13.4
× 100 =
268 2
= 38 %
35 7 7

45 Adda247 | No. 1 APP for Banking & SSC Preparation


Website: bankersadda.com | sscadda.com | store.adda247.com | Email: contact@bankersadda.com
19. (d); Quantity of milk after selling 36 l mixture II. 15y² + 8y + 1 = 0
5
= (180 − 36) × = 80 𝑙 15y² +5y + 3y + 1 = 0
9
5y (3y + 1)+ 1 (3y+ 1) = 0
And quantity of water = (180 − 36) − 80 = 64 𝑙
(3y + 1) (5y + 1)= 0
Now, ratio of milk and water after addition of 6l 1 1
80 8 𝑦 = − ,−
water in mixture = 64+6 = 7 3 5
∴ Relationship between x and y cannot be
Quantity of milk after selling 60 l mixture determined
8
= (150 − 60) × 15 = 48 𝑙
24. (c); I. 15x² – 11x + 2 = 0
48 160
So, required percentage = × 100 = 15x² – 5x – 6x + 2 = 0
90 3
1 5x(3x – 1) – 2 (3x – 1)= 0
= 53 3 %
(3x – 1) (5x – 2)= 0
1 2
20. (b); I. x² – 9x + 18 = 0 𝑥 = 3,5
x² – 6x – 3x + 18 = 0 II. 10y² – 9y + 2 = 0
x(x – 6) –3 (x -6) = 0 10y² – 5y – 4y + 2 = 0
(x – 3) (x – 6)= 0 5y(2y – 1) –2 (2y – 1) = 0
x= 3, 6 (2y – 1) (5y - 2)=0
1 2
II. 5y² – 22y + 24 = 0 y = 2,5
5y² – 10y – 12y + 24 = 0 ∴x≤y
5y(y- 2) –12 (y – 2) = 0 20×270
(y – 2) (5y – 12) = 0 25. (c); + 30 × 2 =? × 2
100
𝑦 = 2,
12 54 + 60 = ? × 2
5 114
∴x>y 2
=?
57 = ?
21. (d); I. 6x² + 11x + 5 = 0
3
6x² + 6x + 5x + 5 = 0 26. (b); √27 × 64 + √64 × 6.25 = √?× 4
6x (x + 1) + 5 (x + 1) = 0 3 × 4 + 8 × 2.5 = √?× 4
32
(x+ 1) (6x +5)= 0 = √?
5 2
𝑥 = 1, −
6
? = 256
II. 2y² + 5y + 3 = 0 24×7
27. (c); 16 × 6– = √?+ 26
2y² + 2y + 3y + 3 = 0 3×2

2y(y+ 1) + 3 (y+ 1) = 0 96 – 4 × 7 = √? + 26
(y+ 1) (2y + 3) = 0 96 – 28 -26 = √?
𝑦 = −1, −
3 96 - 54 = √?
2 ? = 1764
∴𝑥≥𝑦
28. (e); Let initial time = t hours
22. (e); I. x² + 10x + 24 = 0 8
Initial speed = 72 × 9 = 64 kmph
x² +6x + 4x + 24 = 0
ATQ,
x(x + 6) +4 (x+ 6)= 0 25
(x + 4) (x + 6) = 0 64(t) = 72 × (t − )
18
1
x = –4, –6 t = 12 2
II. y² – √625 = 0 So, distance = 64 × 12 2 = 800 km
1

𝑦² = √625
D D 2D
𝑦 2 = 25; 𝑦 = ±5 29. (e); + =
40 50 average speed
∴ Relationship between x and y cannot be 400
Average speed = kmph
determined 9

23. (e); I. 10x² +11x + 1= 0 30. (a); Required probability = 1- probability of getting
sum is 12
10x² + 10x+ x+ 1 = 0 selected observation
10x (x + 1) + 1 (x + 1) = 0 = 1 − total observation
1 35
(x + 1) (10x + 1)= 0 =1 − 36 =36
1
𝑥 = −1, − 10

46 Adda247 | No. 1 APP for Banking & SSC Preparation


Website: bankersadda.com | sscadda.com | store.adda247.com | Email: contact@bankersadda.com
31. (e); Required probability = probability of getting 42. (b); Required difference = (900 + 850 + 500 +
same digit 600) − (750 + 700 + 350 + 500)
6 1
= = = 2850 − 2300 = 550
36 6
43. (a); Required percentage
32. (a); Total no. of letters are 7. (350 + 600 + 500) − (400 + 500 + 350)
No. of vowels and consonant are 3 (i.e. E, E & U) = (400
× 100
+ 500 + 350)
and 4 (i.e. R, Q, S & T) respectively. =
1450 − 1250
× 100 ⟹
200
× 100 = 16%
5!×3! 1250 1250
So, required no. of arrangement = = 360
2
44. (e); Revenue of Keyboard = 850 × 2x = Rs. 1700x
33. (b); Required no. of ways = 10𝐶6 × 8𝐶5 Revenue of Monitor = 500 × 4x = Rs. 2000x
= 210 × 56 = 11760 1700x
So, required percentage = 2000x × 100 = 85%
Solutions (34-38):
45. (d); (A – 2) = 4(B - 2)
Number of Students who like only English = 10
⇒ A – 4B = –6 …(i)
Number of Students who like only Math = 20
and A + 8 = B + 8 + 12
Number of Students who like only Science = 5
⇒ A – B = 12 …(ii)
Number of students who like both English and Math but no
From (i) and (ii)
Science= 5
–3B = –18 ⇒ B = 6 years
Number of students who like both English and Science but
⇒ A = 18 years.
not Math= 10 A 18 3
Number of students who like both Science and Math but ∴ Required ratio = = =
B 6 1
not English = 20
46. (a); Let the initial price be Rs.1
Number of students who like all three subjects = 5 118 59
Number of students who don’t like any subject New Price = 100 = 50
= 150 – (10 + 20 + 5 + 10 + 20 + 5 + 5) Now
59
=150 – 75 = 75 ×x=1
50
50
x=
59
50
∴ Decrease in consumption = 1 –
59
9 900
= × 100 = %
59 59

47. (c); (30-20) % = 5+20


10% = 25
Max marks = 100% = 250
Passing marks = 20% of 250 + 5
34. (b); Required percentage =
10−5
× 100 = 100% = 50 + 5 = 55
5 55
% passing =250 × 100 = 22%
35. (c); Required ratio = 5: 75 = 1: 15
48. (b); Let the salaries of Laxman and Gopal one year
36. (e); Required no. of students = 20 + 5 + 10 =35 before be L₁, G₁ respectively and now L₂, G₂
37. (b); Required ratio = (150 − 50) ∶ 50 = 2 : 1 respectively, Then,
L1 3 L 4 G 2
38. (b); Total number of students who like math as a = , 1= , 1=
G1 4 L2 5 G2 3
6
subject = 50 × 5 = 60 and L₂ + G₂ = 4160
60 Solving these equations, we get L₂ = Rs. 1600
Required percentage = 150 × 100 = 40%
49. (c); Correct average
20+40 60 4 × 75 – 69 – 86 + 96 + 68
39. (d); Required average 2
= 2
= 30 = = 77.25
4
800+350 1150
40. (b); Required ratio = 700+500 = 1200 = 23 : 24 50. (a); Let radius of circle and side of square be r cm and
a cm respectively.
20 10
41. (d); Required no. of items =800 × 100 + 650 × 100 + πr2 = 2464
15
400 × 100 + 350 × 100
12 r = 28 cm
∴ 4x = 28 ⇒ x = 7
= 160 + 65 + 60 + 42 = 327 ∴ Required area = 7 × 7 = 49 sq. cm.

47 Adda247 | No. 1 APP for Banking & SSC Preparation


Website: bankersadda.com | sscadda.com | store.adda247.com | Email: contact@bankersadda.com
Most Important Questions | English Language |
SBI Clerk Prelims 2020
Direction (1-5): Select the phrase/connector (STARTERS) (i) With a requirement of the mandate………
from the given three options which can be used to form a (ii) Since the trustees require……..
single sentence from the two sentences given below, (iii) Because the trustees require………
implying the same meaning as expressed in the statement (a) Only (i)
sentences. (b) Only (ii)
(c) Only (iii)
1. (I) Not a tear needs to be shed for anyone going. (d) All (i), (ii), and (ii)
(II) Universal love and universal confidence stand (e) Both (ii) and (iii)
against the hard winds to which our country may
5. (I) The proposal is not public.
be exposed. (II) Both officials declined to be named.
(i) Until a tear need……… (i) Because the proposal…….
(ii) As long as universal love…… (ii) Declining to be named………
(iii) Unless universal love and………. (iii) Since the proposal……..
(a) Only (i) (a) Only (i)
(b) Only (ii) (b) Only (ii)
(c) Only (iii) (c) Only (iii)
(d) Both (i) and (ii) (d) Both (i) and (iii)
(e) Both (ii) and (iii) (e) Both (ii) and (iii)

2. (I) The Home Ministry has decided to transfer the Directions (6-20): In each of the following sentence, there
Bhinma Koregaon case to NIA. is a blank given. Below the sentences, there are five options
(II) The decision is an unwarranted interference in with a word fitting in each. Fill up the sentence with the
word given in the space so as to make the sentence
the police powers of the State.
grammatically and contextually correct.
(i) The decision of Home Ministry to………..
(ii) Transferring the Bhima Koregaon………. 6. The opening arguments were heard in the Senate trial
(iii) Provided that the Home Ministry….. that will decide whether to remove Donald Trump
(a) Only (i) from power following his ________________ by the House
(b) Only (ii) of Representatives.
(a) epidemic (b) whirling (c) impeachment
(c) Only (iii)
(d) swift (e) None of these.
(d) Both (i) and (iii)
(e) Both (ii) and (iii) 7. Brazilian prosecutors asked a judge to indict Glenn
Greenwald, an American journalist, for helping a
3. (I) The Government has drawn up National group that had ________________the phones of judges and
Infrastructure Pipeline (NIP) Plan. lawyers.
(II) NIP envisages investments of over Rs 102-lakh (a) hacked (b) exciting (c) fertile
crore. (d) altar (e) None of these.
(i) Whenever the Government has………..
8. An annual worldwide survey of chief executives
(ii) In order to envisage investments………
conducted by PWC found that 53% think that global
(iii) Envisaging investments of………… economic growth will ________________over the coming
(a) Only (i) year.
(b) Only (ii) (a) greedy (b) restless (c) contentious
(c) Only (iii) (d) decline (e) None of these.
(d) Both (i) and (ii)
9. Iran threatened to quit the nuclear Non-Proliferation
(e) Both (ii) and (iii)
Treaty if Britain, France and Germany refer it to the
4. (I) The trustees have approached global investment UN Security Council over ________________to the nuclear
bankers. deal they signed in 2015.
(II) The trustees require the mandate to sell their (a) climb (b) search (c) breaches
stake. (d) belittle (e) None of these.

48 Adda247 | No. 1 APP for Banking & SSC Preparation


Website: bankersadda.com | sscadda.com | store.adda247.com | Email: contact@bankersadda.com
10. Vladimir Putin appointed a new cabinet in Russia, 19. The bjp, and Mr Modi in particular, have repeatedly
following his declaration of ________________political won elections by ________________as the protectors of
reforms that could let him stay in power after his term Hindus, who are 80% of the population.
as president expires in 2024. (a) risking (b) beginning (c) dubious
(a) tendency (b) sweeping (c) energy (d) posing (e) None of these.
(d) deceive (e) None of these.
20. In August Mr Modi annulled the statehood of Jammu
11. Honduras ended the ________________of the Mission to & Kashmir, part of a region long ________________over by
Support the Fight Against Corruption and Impunity, India, Pakistan and China.
which is backed by the Organisation of American (a) careless (b) affluent (c) disputed
States. (d) profound (e) None of these.
(a) mandate (b) occasional (c) skilful Directions (21-35): Read each sentence to find out
(d) clean (e) None of these. whether there is any grammatical or idiomatic error in it.
12. The trade war with America hit already-weak The error, if any, will be in one part of the sentence. The
consumer spending and ________________a slowdown in number of that part is the answer. If there is ‘No error’, the
the property market. answer is (e). (Ignore errors of punctuation, if any.)
(a) praise (b) escape (c) pragmatic 21. The relatively new idea of using satellite
(d) exacerbated (e) None of these. (A)/surveillance to document not only the (B)/
13. Vodafone became the latest big name to pull out of the attacks but also identify the perpetrators would
association that is developing Libra, a digital currency (C)/have seemed like the perfect solution to make the
________________by Facebook that has run into horror stop (D)/ No Error (E)
difficulties with financial regulators. (a) A (b) B (c) C
(a) dark (b) devised (c) impudence (d) D (e) No Error
(d) abandon (e) None of these. 22. The statute of limitations on the sexual harassment
14. Doubts were raised about whether the hs2 high-speed allegations (A)/had expired by the time Gonzalez’s
rail project in Britain would go ahead, after the letter (B)/sent and the city attorney’s office
estimate for its cost ________________to as much as (C)/declined to investigate the claims (D)/ No Error
£106bn ($140bn). (E)
(a) candid (b) thrifty (c) soared (a) A (b) B (c) C
(d) leisure (e) None of these. (d) D (e) No Error

15. The deliberate and sustained persecution of one of 23. The minister then (A)/asked me where (B)/had I
them constitutes an ________________threat against all— been when (C)/he was being heckled (D)/ No Error
and so puts the political system at risk. (E)
(a) forget (b) implicit (c) mitigate (a) A (b) B (c) C
(d) chase (e) None of these. (d) D (e) No Error

16. The sad truth is that Mr Modi and the BJP are likely to 24. Samuda, seemingly annoyed (A)/by the continued
________________politically by creating divisions over (B)/delay, insisted to make (C)/his statement and
religion and national identity. refused to yield (D)/ No Error (E)
(a) benefit (b) bully (c) cowardly (a) A (b) B (c) C
(d) profane (e) None of these. (d) D (e) No Error

17. The Chief Justice orally observed that the plea of a 25. Director has revealed that one of (A)/the key reasons
man, whose ________________is scheduled for February why he was taken (B)/that decision is to have
1, ought to be given top priority. (C)/ample time and focus on his company. (D)/ No
(a) pleasant (b) anarchy (c) deviation Error (E)
(d) execution (e) None of these. (a) A (b) B (c) C
(d) D (e) No Error
18. Mukesh has asked the court to call for his medical
records, dating back to the time of his arrest and 26. Inequality is growing for much than (A)/70 per cent
________________, which were placed before the of the global population,(B)/ exacerbating the risks of
President for a decision on his mercy plea. divisions and (C)/hampering economic and social
development (D)/ No Error (E)
(a) immediate (b) incarceration (c) confusion
(a) A (b) B (c) C
(d) oblivious (e) None of these.
(d) D (e) No Error

49 Adda247 | No. 1 APP for Banking & SSC Preparation


Website: bankersadda.com | sscadda.com | store.adda247.com | Email: contact@bankersadda.com
27. Every year, our industry becomes more (A)/and more 36. By working part-time and looking after her kids two
aware of issues we face (B)/as a community and days a week she managed to get the best of both
prioritizes taking action (C)/to secure a better future worlds.
for generations to come (D)/ No Error (E) (a) to be at two places
(a) A (b) B (c) C (b) to cheat someone
(d) D (e) No Error (c) to gain all the advantages
(d) to make the next decision
28. We must not withdrew (A)/attention and resources
(e) to fight justice
(B)/from species elsewhere (C)/ that need saving
(D)/ No Error (E) 37. I am still too early in the process to get up on my
(a) A (b) B (c) C soapbox and proclaim that intermittent fasting is the
(d) D (e) No Error best thing since sliced bread when it comes to
weight management.
29. Some netizens even (A)/ suggested Malaysian
(a) best part of the world
restaurants (B)/in London that serves good (C)/
(b) best food tasted in a while
versions of the dish (D)/ No Error (E)
(c) being happy with a person
(a) A (b) B (c) C
(d) A good invention or innovation
(d) D (e) No Error
(e) something good that isn’t recognized at first
30. The new policy stipulates that if (A)/employees give
38. Failing to do your research before pursuing a form of
off smoking for (B)/ a year then they will be entitled
credit or a lending option can lead to you biting off
(C)/to the four days extra holiday as well (D)/ No
more than you can chew financially.
Error (E)
(a) to spend more than you earn
(a) A (b) B (c) C
(b) to take on a task that is way to big
(d) D (e) No Error
(c) to take multiple tasks to perform
31. The world will be a very (A)/different place if we (d) to buy something very expensive
don’t (B)/listen very careful to the voices (C)/of (e) to complain about a loss from the past
climate movement activists (D)/ No Error (E)
39. When our galaxy collides with Andromeda in about 4
(a) A (b) B (c) C
billion years, there will be no crying over spilt milk.
(d) D (e) No Error
(a) to fail in something
32. Chew claims that the Xiaomi‘s plan for (A)/2020 (b) to be happy when a person leaves
includes a big emphasis with (B)/ Western Europe (c) to face additional losses
due to them (C)/gaining plenty of traction there (D)/ (d) to complain about a loss from the past
No Error (E) (e) to compensate the earlier losses
(a) A (b) B (c) C
40. When he started asking too many questions of his
(d) D (e) No Error
neighbours about their whereabouts during the
33. Except for (A)/you and I (B)/everyone have(C)/ weekend, they warned him that curiosity killed the
brought a gift for me (D)/No Error(E). cat.
(a) A (b) B (c) C (a) when something is done badly
(d) D (e) E (b) to present a counter argument
(c) believe someone’s statement without proof
34. We have filed (A)/ a complaint against the (B)/
(d) to go to bed
contractor whom (C)/we hired last month. (D)/No
(e) to be inquisitive leading you into an unpleasant
Error(E)
situation.
(a) A (b) B (c) C
(d) D (e) E Direction (41-45): There are two different sentences with
a blank space in each question. Choose the word from the
35. Japan is now (A) /very different than (B) /what it was
given options which fits into both the blanks appropriately
(C)/ twenty years ago (D)/ No Error(E)
adding a proper and logical meaning to the sentences.
(a) A (b) B (c) C
(d) D (e) E 41. (1) A sense of dignity and self-respect goes missing in
the national trait of ________________ some so-called
Direction (36-40): The sentences given below consist of ‘celebrity’.
an idiom given in bold. From the given five alternatives (2) When we reached the summit, we could hear the
identify the most suitable option reflecting the meaning of magpies calling out, but, to do them justice, they
the idiom. were not _______________ us then.

50 Adda247 | No. 1 APP for Banking & SSC Preparation


Website: bankersadda.com | sscadda.com | store.adda247.com | Email: contact@bankersadda.com
(a) mobbing (b) retreating (c) inserting E. An IMF blog post points out that European and
(d) shunned (e) compressing Japanese banks, and not American banks, dominate
international banks’ lending in dollars.
42. (1) Gender stereotyping in the classroom particularly
F. These banks cannot easily tap the dollars deposited at
among younger children, has an adverse impact
their American subsidiaries to fund themselves.
on the learning abilities and _______________ of both
G. In terms of economic strength, the eurozone looks
boys and girls.
brittle.
(2) Social institutions and mores were adapted to
H. While America’s cyclical expansion is going strong in
reflect the preferences, inclinations, values, goals
its 10th year, eurozone recovery has fizzled out.
and _______________ of the new elite.
(a) intention (b) regulation (c) worldview 46. Which sentence should be the FOURTH in the
(d) authority (e) protection paragraph?
(a) H (b) A (c) B
43. (1) The ________________ of cryptocurrencies has been
(d) D (e) C
growing rapidly in India due to benefits in terms
of anonymity, accessibility, speed, and cost. 47. Which sentence should be the SEVENTH in the
(2) His ________________ with the people was paragraph?
unbounded, but in the midst of it all he begged to (a) F (b) B (c) C
be removed to London. (d) D (e) E
(a) confidence (b) popularity (c) practice
48. Which sentence should be the FIRST in the
(d) consideration (e) custom
paragraph?
44. (1) The Supreme Court rightly notes that the (a) G (b) H (c) A
tribunal’s view ignores the basic principle ________ (d) D (e) B
to drinking water.
49. Which sentence should be the SECOND in the
(2) It was a list of questions, all of them apparently
paragraph?
_______________ to her mental health.
(a) A (b) B (c) H
(a) assessing (b) reckon (c) concerned
(d) D (e) C
(d) contemplating (e) pertaining
50. Which sentence should be the THIRD in the
45. (1) With the hype surrounding cryptocurrencies
paragraph?
continuing to grow, more people are hearing
(a) A (b) H (c) C
about the _______________ technology known as
(d) D (e) G
blockchain.
(2) Although the architecture of the blockchain is far
more complex than these aggregators, the
___________ principle is not that different.
(a) subtle (b) apparent (c) Underlying
(d) ample (e) substantial
Directions (46-50): Rearrange the following EIGHT
sentences (A), (B), (C), (D), (E), (F), (G) and (H) in the
proper sequence to form a meaningful paragraph then
answer the questions given below them.
A. American banks have passed the stress test that the
Federal Reserve conducts.
B. European banks remain fragile.
C. But, when the booms end, banks will be worse off.
Eurozone banks have another and bigger
vulnerability.
D. That is a reason why the European Central Bank is
forced to keep interest rates so low and why it risks
keeping alive real estate booms in many European
cities.

51 Adda247 | No. 1 APP for Banking & SSC Preparation


Website: bankersadda.com | sscadda.com | store.adda247.com | Email: contact@bankersadda.com
Solutions
1. (b); Only starter (II) can be used to form a single 5. (d); Both (i) and (iii) starters can be used to form a
sentence from the two given sentences, implying single sentence from the two given sentences,
the same meaning as expressed in the statement implying the same meaning as expressed in the
sentences. The sentence thus formed is, statement sentences. The sentences thus formed
As long as universal love and universal are:
confidence stand against the hard winds to (i) Because the proposal is not public, both
which our country may be exposed, not a tear the officials declined to be named.
needs to be shed for anyone going. (iii) Since the proposal is not public, both the
Hence, option (b) is the right answer choice. officials declined to be named.
2. (d); Both (i) and (iii) starters can be used to form a Hence, option (d) is the right answer choice.
single sentence from the two given sentences, 6. (c); In the given sentence, the word ‘impeachment’
implying the same meaning as expressed in the makes the sentence grammatically as well as
statement sentences. The sentences thus formed contextually correct. While all other options are
are:
grammatically and contextually wrong.
(i) The decision of Home Ministry to transfer
Whirling means characterized by rapid
the Bhinma Koregaon case to NIA is an
movement round and round.
unwarranted interference in the police
Swift means happening quickly or promptly.
powers of the State.
(iii)Provided that the Home Ministry has 7. (a); In the given sentence, the word ‘hacked’ makes
decided to transfer the Bhinma Koregaon the sentence grammatically as well as
case to NIA, it is an unwarranted interference contextually correct. While all other options are
in the police powers of the State. grammatically and contextually wrong.
Hence, option (d) is the right answer choice.
8. (d); In the given sentence, the word ‘decline’ makes
3. (e); Both (ii) and (iii) starters can be used to form a the sentence grammatically as well as
single sentence from the two given sentences, contextually correct. While all other options are
implying the same meaning as expressed in the grammatically and contextually wrong.
statement sentences. The sentences thus formed Contentious means ‘causing or likely to cause an
are: argument; controversial’.
(ii) In order to envisage investments of over
Rs 102-lakh crore, the government has 9. (c); In the given sentence, ‘breaches’ make the
drawn up National Infrastructure Pipeline sentence grammatically as well as contextually
(NIP) Plan. correct. While all other options are
(iii) Envisaging investments of over Rs 102- grammatically and contextually wrong.
lakh crore, the government has drawn up
10. (b); In the given sentence, the word ‘sweeping’
National Infrastructure Pipeline (NIP) Plan.
Hence, option (e) is the right answer choice. makes the sentence grammatically as well as
contextually correct. While all other options are
4. (d); All (i), (ii), and (iii) starters can be used to form grammatically and contextually wrong.
a single sentence from the two given sentences,
implying the same meaning as expressed in the 11. (a); In the given sentence, the word ‘mandate’
statement sentences. The sentences thus formed makes the sentence grammatically as well as
are: contextually correct. While all other options are
(i) With a requirement of the mandate to sell grammatically and contextually wrong.
their stake, the trustees have approached 12. (d); In the given sentence, the word ‘exacerbated’
global investment bankers. makes the sentence grammatically as well as
(ii) Since the trustees require the mandate to contextually correct. While all other options are
sell their stakes, they have approached grammatically and contextually wrong.
global investment bankers.
(iii) Because the trustees require the 13. (b); In the given sentence, the word ‘devised’ makes
mandate to sell their stakes, they have the sentence grammatically as well as
approached global investment bankers. contextually correct. While all other options are
Hence, option (d) is the right answer choice. grammatically and contextually wrong.

52 Adda247 | No. 1 APP for Banking & SSC Preparation


Website: bankersadda.com | sscadda.com | store.adda247.com | Email: contact@bankersadda.com
14. (c); In the given sentence, the word ‘soared’ makes 25. (b); The error lies in part (B) of the sentence. ‘Was
the sentence grammatically as well as taken’ will be replaced by ‘took’ as in active
contextually correct. While all other options are voice, subject+verb is used while in passive
grammatically and contextually wrong. voice, subject+to be+V3 is used. Hence, option
(b) is the right answer choice.
15. (b); In the given sentence, the word ‘implicit’ makes
the sentence grammatically as well as 26. (a); The error lies in part (A). ‘more’ will be used
contextually correct. While all other options are instead of ‘much’ because before adjective of
grammatically and contextually wrong. comparative degree is used before ‘than’.
While for ‘many or much’, comparative degree
16. (a); In the given sentence, the word ‘benefit’ makes is ‘more’ and superlative degree is ‘most’.
the sentence grammatically as well as Hence, option (a) is the right answer choice.
contextually correct. While all other options are
grammatically and contextually wrong. 27. (b); The error lies in part (B) of the sentence. ‘The’
will be used before ‘issues’ because ‘issues’ is
17. (d); In the given sentence, the word ‘execution’ definite here. Hence, option (b) is the right
makes the sentence grammatically as well as answer choice.
contextually correct. While all other options are
grammatically and contextually wrong. 28. (a); The error lies in part (A) of the sentence.
‘Withdrew’ will be replaced by ‘withdraw’
18. (b); In the given sentence, the word ‘incarceration’ because ‘must’ is a model auxiliary that always
makes the sentence grammatically as well as takes first form of verb with it. Hence, option (a)
contextually correct. While all other options are is the right answer choice.
grammatically and contextually wrong.
29. (c); The error lies in part (C) of the sentence. ‘Serves’
19. (d); In the given sentence, the word ‘posing’ makes will be replaced by ‘Serve’ as ‘Malaysian
the sentence grammatically as well as restaurants in London’ is a plural subject.
contextually correct. While all other options are Plural verb is used with plural subject. Hence,
grammatically and contextually wrong. option (c) is the right answer choice.
20. (c); In the given sentence, the word ‘disputed’ 30. (b); The error lies in part (B) of the sentence. ‘Give
makes the sentence grammatically as well as off’ will be replaced by ‘Give up’ as it means to
contextually correct. While all other options are leave something or a habit. ‘Give off’ means ‘to
grammatically and contextually wrong. send out as a branch’ or ‘emit an unpleasant
smell’. Hence, option (b) is the right answer
21. (b); Here, the error lies in part (B) of the given choice.
sentence. Not only will be used before
document because but also is used before 31. (c); The error lies in part (C) of the sentence.
identify. Not only-but also/ either-or/ ‘Carefully’ will replace ‘careful’ because careful
neither-nor etc. are used to connect two is an adjective while carefully is an adverb.
subject/objects/verbs/gerunds etc. Hence, Hence, option (c) is the right answer choice.
option (b) is the right answer choice. 32. (b); The error lies in part (B) of the sentence. ‘On’
22. (c); The error lies in part (C). Was will be used will replace ‘with’ because with ‘emphasis’
before Sent. Hence, option (c) is the right preposition ‘on’ is used. Hence, option (b) is the
answer choice. right answer choice

23. (c); The error lies in part (C) of the sentence. ‘had I 33. (b); In the second part of the sentence “I” must be
been’ will be replaced by ‘I had been’ because replaced with “me” because of the incorrect
the given sentence is in indirect narration. In usage of the preposition. All other options are
indirect narration, we do not use interrogative correct. So option (B) is the correct choice.
sentence but assertive sentence (subject+verb) 34. (c); In the third part of the sentence “whom” must be
is used. Hence, option (c) is the right answer replaced by “who” as who is used for the person.
choice. Here it refers to the contractor. All other options
are correct. So option(c) is correct.
24. (c); The error lies in part (C) of the sentence. ‘to
make’ will be replaced by ‘on making’, because 35. (b); In the second part of the sentence “than” must be
the standard format is ‘insist+on+object’ OR replaced with “from” because different is
‘insist+on+V(ing)’. Hence, option (c) is the followed with “from”. All other parts of the
right answer choice. sentence are correct. So option (b) is correct.

53 Adda247 | No. 1 APP for Banking & SSC Preparation


Website: bankersadda.com | sscadda.com | store.adda247.com | Email: contact@bankersadda.com
36. (c); “To get the best of both worlds” means a win- Intention means a thing intended; an aim or
win situation. A situation in which one can plan.
get the advantages of two different or Regulation means a rule or directive made and
contrasting things at the same time. A maintained by an authority.
situation where one can enjoy the benefit of
43. (b); The word “popularity” means the state or
two different opportunities. Hence, option (c)
becomes the most viable answer choice. condition of being liked, admired, or
supported by many people. Thus, the word fits
37. (d); “Best thing since sliced bread” means perfectly into both the blanks and gives a
something that is too fine; an excellent meaningful sense to both the sentences which is
person or thing; a new invention that is likely not the case with the other given words. Hence
to improve people's lives significantly; option (b) is the correct choice.
something that is thought to be very good. Confidence means the feeling or belief that one
Hence, option (d) is the most suitable answer can have faith in or rely on someone or
choice. something.
38. (b); “Biting off more than you can chew” means to Consideration means careful thought, typically
try to do too much; to take on or attempt over a period of time.
more than one is capable of doing. Hence, Custom means a thing that one does habitually.
option (b) expresses the most suitable meaning 44. (e); Option (e) is the correct choice for the given
of the given idiom.
question.
39. (d); “To cry over spilt milk” means to be upset Pertaining to means having to do with;
about things that have already been done; to belonging to
cry about past events that cannot be undone; Reckon means establish by calculation.
to dwell pointlessly about a mistake that can
45. (c); Option (c) is the correct choice for the given
no longer be changed; to be upset about past
misfortunes; to feel sorry about something sentences.
that has already happened. Hence, option (d) Underlying is a present participle of underlie.
perfectly satisfies the meaning of the idiom. It means real but not immediately obvious.
Ample means enough or more than enough;
40. (e); “curiosity killed the cat” means too much plentiful.
curiosity can lead to dangerous situations; Substantial means of considerable importance,
being too inquisitive can get you into trouble; size, or worth.
a prying behaviour can be harmful; used to
warn someone not to ask too many questions 46. (c); GHABDCEF is the correct sequence of the given
about something. Hence, option (e) best sentences.
substitutes the meaning for the given idiom. G is the first sentence as the economic strength
or fragility of Eurozone is under discussion in
41. (a); The word “mobbing” means crowding round the given passage. H is suitable for the second
(someone) or into (a place) in an unruly way. sentence as Eurozone is compared with America
Thus, the word fits perfectly into both the blanks
which is going strong in comparison to
and gives a meaningful sense to both the
Eurozone. The word “while” produces
sentences which is not the case with the other
contrasting tone which was the requirement of
given words. Hence option (a) is the correct
the passage. A-B is the third combination which
choice.
comes after G-H as the comparison is continued
Retreat means move back or withdraw.
Insert means place, fit, or push (something) into in these sentences. D is the fifth sentence as it is
something else. mentioned that “That is a reason why the
Shun means persistently avoid, ignore, or reject European Central Bank is forced to keep interest
(someone or something) through antipathy or rates so low”, the reason was the fragility of
caution. European banks. Thus, there is a connection
between B and D. C is the obvious choice for the
42. (c); The word “worldview” means a particular next statement as because of the use of the word
philosophy of life or conception of the world. “boom” which was mentioned previously in
Thus, the word fits perfectly into both the blanks statement D. It is mentioned in C that next
and gives a meaningful sense to both the vulnerability awaits Eurozone, the vulnerability
sentences which is not the case with the other is then discussed in statement E. The
given words. Hence option (c) is the correct vulnerability is then elaborated in statement F.
choice.
54 Adda247 | No. 1 APP for Banking & SSC Preparation
Website: bankersadda.com | sscadda.com | store.adda247.com | Email: contact@bankersadda.com
47. (e); GHABDCEF is the correct sequence of the given contrasting tone which was the requirement of
sentences. the passage. A-B is the third combination which
G is the first sentence as the economic strength comes after G-H as the comparison is continued
or fragility of Eurozone is under discussion in the in these sentences. D is the fifth sentence as it is
given passage. H is suitable for the second mentioned that “That is a reason why the
sentence as Eurozone is compared with America European Central Bank is forced to keep interest
which is going strong in comparison to rates so low”, the reason was the fragility of
Eurozone. The word “while” produces European banks. Thus, there is a connection
contrasting tone which was the requirement of between B and D. C is the obvious choice for the
the passage. A-B is the third combination which next statement as because of the use of the word
comes after G-H as the comparison is continued “boom” which was mentioned previously in
in these sentences. D is the fifth sentence as it is statement D. It is mentioned in C that next
mentioned that “That is a reason why the vulnerability awaits Eurozone, the vulnerability
European Central Bank is forced to keep interest is then discussed in statement E. The
rates so low”, the reason was the fragility of vulnerability is then elaborated in statement F.
European banks. Thus, there is a connection
50. (a); GHABDCEF is the correct sequence of the given
between B and D. C is the obvious choice for the
sentences.
next statement as because of the use of the word
G is the first sentence as the economic strength
“boom” which was mentioned previously in
or fragility of Eurozone is under discussion in the
statement D. It is mentioned in C that next
given passage. H is suitable for the second
vulnerability awaits Eurozone, the vulnerability
sentence as Eurozone is compared with America
is then discussed in statement E. The
which is going strong in comparison to
vulnerability is then elaborated in statement F.
Eurozone. The word “while” produces
48. (a); GHABDCEF is the correct sequence of the given contrasting tone which was the requirement of
sentences. the passage. A-B is the third combination which
G is the first sentence as the economic strength comes after G-H as the comparison is continued
or fragility of Eurozone is under discussion in the in these sentences. D is the fifth sentence as it is
given passage. H is suitable for the second mentioned that “That is a reason why the
sentence as Eurozone is compared with America European Central Bank is forced to keep interest
which is going strong in comparison to rates so low”, the reason was the fragility of
Eurozone. The word “while” produces European banks. Thus, there is a connection
contrasting tone which was the requirement of between B and D. C is the obvious choice for the
the passage. A-B is the third combination which next statement as because of the use of the word
comes after G-H as the comparison is continued “boom” which was mentioned previously in
in these sentences. D is the fifth sentence as it is statement D. It is mentioned in C that next
mentioned that “That is a reason why the vulnerability awaits Eurozone, the vulnerability
European Central Bank is forced to keep interest is then discussed in statement E. The
rates so low”, the reason was the fragility of vulnerability is then elaborated in statement F.
European banks. Thus, there is a connection
between B and D. C is the obvious choice for the
next statement as because of the use of the word
“boom” which was mentioned previously in
statement D. It is mentioned in C that next
vulnerability awaits Eurozone, the vulnerability
is then discussed in statement E. The
vulnerability is then elaborated in statement F.
49. (c); GHABDCEF is the correct sequence of the given
sentences.
G is the first sentence as the economic strength
or fragility of Eurozone is under discussion in the
given passage. H is suitable for the second
sentence as Eurozone is compared with America
which is going strong in comparison to
Eurozone. The word “while” produces

55 Adda247 | No. 1 APP for Banking & SSC Preparation


Website: bankersadda.com | sscadda.com | store.adda247.com | Email: contact@bankersadda.com
Most Important Questions | Reasoning Ability |
SBI Clerk Prelims 2020
Directions (1-5): Study the following arrangement 6. Statements: G=W>Y≥U>P=O<L=C
carefully and answer the questions given below. Conclusion I: W>P II: C>U
Some people are sitting in two parallel rows facing each 7. Statements: X=H<I≥K≥L=M>N≤S
other. A, B, C, D, E, F and G are sitting in row 1 facing north Conclusion I: I>L II: I>M
and P, Q, R, S, T, U and V are sitting in row 2 facing south 8. Statements: W=V<B>T=Y<Z>M=Q
(not necessarily in the same order). Three persons sit Conclusion I: B<Z II: T>W
between B and C and one of them sits at an end. Q sits
fourth to the right of T. P sits to the immediate left of U, who 9. Statements: Q=N≥M≤J>L>I>U=Y
sits at the middle of the row. A is not a neighbor of B. E sits Conclusion I: Q>M II: J>Y
to the immediate right of the one who faces P. D faces the 10. Statements: T=Q>P<L>K≥I≥M=A
one who sits second to the left of V. S faces G. R faces the Conclusion I: K>A II: A=K
one who sits to the immediate right of A. T does not sit at
Directions (11-15): These questions are based on the
any end.
following arrangement. Study the arrangement carefully to
1. Who among the following sits second to the left of the answer these questions.
one who is facing T?
A%T31H@7UOEL8GP#UJ$R2K*CMV9A6
(a) F (b) E (c) C
&SZ
(d) A (e) None of these
11. How many such symbols are there in the above
2. How many persons sit between C and the one who arrangement, each of which is immediately followed
faces P? by a vowel?
(a) None (b) One (c) Two (a) One (b) None (c) Two
(d) Three (e) More than three (d) Three (e) More than three
3. Who among the following faces the person who sits 12. Which of the following element is exactly in the
diagonally opposite to S? middle of 3rd element from the right end and the 10th
(a) G (b) C (c) V element from the left end of the arrangement?
(d) Q (e) T (a) 2 (b) R (c) $
4. Who among the following pair sits at the extreme end (d) K (e) None of these
of the row? 13. How many such symbols are there in the above
(a) G, B (b) B, S (c) V, C arrangement, each of which is immediately preceded
(d) S, E (e) None of these by a number and also immediately followed by a
consonant?
5. What is the position of P with respect to Q?
(a) None (b) One (c) Two
(a) third to the left (b) third to the
(d) Three (e) More than three
right (c) second to the left
(d) immediate right (e) immediate left 14. How many such vowels are there in the above
arrangement each of which is immediately followed
Direction (6-10): In these questions, relationship
by a number and also immediately preceded by a
between different elements is show in the statements. The
consonant?
statements are followed by conclusions. Study the
(a) None (b) One (c) Two
conclusions based on the given statements and select the
appropriate answer: (d) Three (e) None of these
(a) If only conclusion I follows. 15. What should come in the place of question mark (?) in
(b) If only conclusion II follows. the following series based on above arrangement?
(c) If either conclusion I or II follows OEU G8P J$U K*?
(d) If neither conclusion I nor II follows. (a) C (b) M (c) V
(e) If both conclusions I and II follow. (d) 2 (e) None of these

56 Adda247 | No. 1 APP for Banking & SSC Preparation


Website: bankersadda.com | sscadda.com | store.adda247.com | Email: contact@bankersadda.com
Directions (16-20): In each of the questions given below Conclusions:
some statements are followed by some conclusions. You I. All clock can be number
have to take the given statements to be true even if they II. Some number are text
seem to be at variance from commonly known facts. Read III. Some digit are text
all the conclusions and then decide which of the given (a) Only II follows
conclusions logically follows from the given statements (b) Only I follows
disregarding commonly known facts. (c) Only II and III follow
16. Statements: (d) Only III follows
Some studio are hall (e) None follows
All hall are room
Only room is bag 20. Statements:
Conclusion: All black are white
I. Some bag are hall All grey are white
II. Some studio are room Only white is Pink
III. All Studio can be hall Conclusions:
(a) Only I is true I. Some Black can be grey
(b) Only II is true II. Some pink can be grey
(c) Both I and II is true III. All White can be Black
(d) Only III is true (a) Only I follows
(e) None of these (b) Only II and III follow
17. Statements: (c) Either I and III follow
All rain are rays (d) Only I and III follow
All rays are speed (e) None of these
Some speed are thrill
21. How many such pairs of letters (both forward and
Conclusion:
backward direction) are there in the word
I. Some rain are thrill
II. All speed are rays ‘INDEFINITE’ each of which has as many letters
III. No thrill is Rain between them in the word as they have in the English
(a) Both I and III is true alphabet?
(b) Both I and II is true (a) Three (b) Seven (c) Two
(c) Only I is true (d) Four (e) One
(d) Only II is true
22. If in a certain language ‘Black’ is called ‘White’, ‘white’
(e) None of these
is called ‘Blue’; ‘Blue’ is called ‘Pink’, ‘Pink’ is called
18. Statements: ‘Grey’; ‘Grey’ is called ‘Yellow’ and ‘Yellow’ is called
Some Clerk are PO ‘Purple’; then what is the colour of Sky?
All PO are AGM (a) Blue (b) Pink (c) White
All AGM are DGM (d) Black (e) Grey
Conclusion:
I. Some AGM are Clerk 23. In a certain code, MISTAKEN is written as TSIMNEKA.
II. All Clerk are DGM How is SURROUND written in that code?
III. All DGM can be Clerk (a) RRUSDNUO (b) RRSUDNUO (c) SRUDRNOU
(a) Both II and III is true (d) RSRUDONU (e) None of these
(b) Only III is true
(c) Both I and II is true 24. Aman cycled 20 km South direction from his home,
(d) Both I and III is true then turned left and cycled 18 km, then turned right
(e) None of these and cycled 12 km, then turned right and cycled 18 km
and then stopped. What is the shortest distance
19. Statements:
between his final position and his home?
Some number are digit
(a) 22 km (b) 32 km (c) 29 km
All digit are clock
(d) 30 km (e) 33 km
Only a few clock are text

57 Adda247 | No. 1 APP for Banking & SSC Preparation


Website: bankersadda.com | sscadda.com | store.adda247.com | Email: contact@bankersadda.com
25. A man travels 4 km towards North, then he travels 6 Directions (33-35): Study the information carefully and
km towards East direction and then again he travels 4 answer the questions given below.
km towards North. Then in which direction is the man
Seven persons A, B, C, D, E, F and G are sitting in a circular
from his starting point?
table Some of them facing towards center and some of
(a) North (b) South east (c) South
them facing away from center but not necessary in same
(d) East (e) North East
order.
Directions (26-29): Study the information carefully and D sits third to the right of B. E sits immediate left of D. Two
answer the questions given below. persons sit between B and G. A is immediate neighbor of C.
A sits third to the right of F. Immediate neighbor of G face
Seven persons P, Q, R, S, T, U and V are different heights. P
is taller than more than three persons. R is taller than Q and opposite direction to each other. G face towards the center.
shorter than U. V is taller than S, who is taller than R. Q is Neither G nor C face same direction as D. Not more than
not the shortest person. V is not the tallest person. The four person face towards center.
height of shortest person is 157cm. V is taller than U. 33. Who among the following person sits second to the
26. Who among the following person is third tallest? right of C?
(a) U (b) S (c) R (a) F (b) D (c) E
(d) V (e) Can’t be determined (d) G (e) None of these

27. Who among the following is the shortest person? 34. How many persons sit between A and E, when
(a) Q (b) T (c) R counted to the right of E?
(d) S (e) None of these (a) Two (b) Three (c) One
(d) Four (e) None of these
28. Who among the following person is tallest?
(a) P (b) U (c) V 35. How many persons face outside the center?
(d) Q (e) None of these (a) Four (b) Two (c) Three
(d) Five (e) None of these
29. If height of R is 167cm what may be the height of Q?
(a) 160cm (b) 169cm (c) 151cm Directions (36-40): Study the information carefully and
(d) 155cm (e) None of these answer the questions given below.

Directions (30-32): Study the following information Three friends’ covers a certain distance from starting point
carefully and answer the question given below- K. Ram starts walking towards east from point K and walks
5m to reach point Y. From point Y, he walks 8m north
Seven members of a family are living in a house. There are direction and then he takes a left turn and walks 9m to
three married couples and three generations in this family. reach at point M. Ramesh starts walking in west direction
D is grandmother of C, who is niece of L. A has only two and walks 8m to reach point V. from point V he takes right
children. B is sister-in-law of K. F is mother of C, who has turn and walks 5m to reach point O. From point O he starts
no siblings. F is not sister of L. walking in east direction and walks 9m to reach at point B.
30. If E is sister of B, then how L is related to E? Manoj start walking in north direction and walks 3m and
(a) Sister reach at point U then takes right turn and walks certain
(b) Brother distance to reach at point H from point H, he starts walking
(c) Brother-in-law in north direction to reach at point B.
(d) Father 36. What is the distance between point U and H?
(e) None of these (a) 2m (b) 3m (c) 4m
31. How A is related to F? (d) 5m (e) None of these
(a) Father 37. In which direction point O with respect to point U?
(b) Father-in-law
(a) North (b) South (c) North-west
(c) Son (d) South-east (e) None of these
(d) Brother-in-law
(e) None of these 38. What is the shortest distance between point M and
point K?
32. If M is son of L, then how M is related to D?
(a) 2√5m (b) 4√5m (c) √60m
(a) Grandfather (b) Nephew (c) Grandson
(d) 9m (e) None of these
(d) Son (e) None of these

58 Adda247 | No. 1 APP for Banking & SSC Preparation


Website: bankersadda.com | sscadda.com | store.adda247.com | Email: contact@bankersadda.com
39. If point S is 5m south of point M then in which (c) One box is placed between C and G
direction point S with respect to point H? (d) More than three boxes are placed above box G
(a) North west (b) Southwest (c) East (e) Both (b) and (d)
(d) West (e) None of these
45. Which of the following box is placed just below box C?
40. In which direction point O with respect to the starting (a) D (b) H (c) G
point? (d) A (e) None of these
(a) North (b) South-east (c) North-west
(d) South (e) None of these Directions (46-50): Study the information and answer the
following questions:
Directions (41-45): Study the information carefully and
answer the questions given below. In a certain code language,
‘strike publish hype place’ is written as ‘ er ol mz et’,
Nine boxes are placed one above another in a stack. Four ‘road prove highway health’ is written as ‘ ca om zc bt’,
boxes are placed between F and H. More than Four boxes ‘dream place minor highway’ is written as ‘ mz bt hx sv’,
are placed above the box H. Two boxes are placed between ‘road length dream hype’ is written as ‘ hx ol nc ca’,
box B and C, which is placed above box H. There are three
boxes are placed between box E and box D, which is placed 46. What is the possible code for ‘publish search minor’ in
just above box F. More than three boxes are placed the given code language?
between box C and box A, which is placed above box B. (a) sv er bt (b) er sv nc (c) et sv ca
More than two boxes are placed between box G and box K, (d) sv et ya (e) et sv om
which is not placed above box C.
47. What is the code for ‘road place’ in the given code
41. Which of the following box is placed just above box B? language?
(a) G (b) H (c) A (a) ca bt (b) mz et (c) mz ca
(d) F (e) None of these (d) ca er (e) None of these
42. Which of the following box is placed in the bottom of 48. What is the code for ‘hype’ in the given code language?
the stack? (a) et (b) bt (c) ol
(a) B (b) K (c) H (d) er (e) None of these
(d) C (e) None of these
49. What is the code for ‘length’ in the given code
43. How Many boxes are placed between box A and Box language?
H? (a) hx (b) ol (c) nc
(a) Four (b) Three (c) More than four (d) ca (e) None of these
(d) One (e) None
50. What is the code for ‘health’ in the given code
44. Which of the following statement is false about box G?
language?
(a) Box G is placed just above box E
(a) zc (b) om (c) ca
(b) Not more than two boxes are placed between box
(d) bt (e) Cannot determined
A and G

59 Adda247 | No. 1 APP for Banking & SSC Preparation


Website: bankersadda.com | sscadda.com | store.adda247.com | Email: contact@bankersadda.com
Solutions
Solutions (1-5): 20. (a);

21. (b); 22. (b); 23. (a);


24. (b);
1. (a); 2. (d); 3. (c);
4. (c); 5. (a);
Solutions (6-10):
6. (a); I: W>P(True) II: C>U(False)
7. (d); I: I>L(False) II: I>M (False)
8. (d); I: B>Z (false) II: T>W (False)
9. (b); I: Q>M(False) II: J>Y(True)
25. (e);
10. (c); I: K>A(False) II: A=K (False)
11. (a); # U
12. (b); 13. (b); 14. (a);
15. (d);
16. (e);

Solutions (26-29):
P>V>U/S>S/U>R>Q>T(157cm)
26. (e);
17. (e); 27. (b);
28. (a);
29. (a);
Solutions (30-32):

18. (d);

19. (b);
30. (c);
31. (b);
32. (c);

60 Adda247 | No. 1 APP for Banking & SSC Preparation


Website: bankersadda.com | sscadda.com | store.adda247.com | Email: contact@bankersadda.com
Solutions (33-35): 41. (d); 42. (b); 43. (c);
44. (b); 45. (b);
Solutions (46-50):
Word Code
Strike/publish er/et
Hype ol
Place mz
Road ca
33. (d); 34. (a); 35. (c); Prove/health om/zc
Solutions (36-40): Highway bt
Minor sv
Dream hx
Length nc

46. (d); 47. (c); 48. (c);


49. (c); 50. (e);

36. (e); 37. (c); 38. (b);


39. (d); 40. (c);
Solutions (41-45):
Boxes
A
D
F
B
G
E
C
H
K

61 Adda247 | No. 1 APP for Banking & SSC Preparation


Website: bankersadda.com | sscadda.com | store.adda247.com | Email: contact@bankersadda.com

You might also like